120
Model Question Papers (with answers) Price : ` 40/- SRI GANGA PUBLICATIONS (A UNIT OF SHYAMALA GROUP) Corporate Office : No. 1, Sugar Mill Colony, Salai Kumaran illam, Madurai Road, Tirunelveli - 627001. Phone : 0462 - 233 8899 / 233 8484 Mobile : 94431 58484 / 95978 39822 Email : [email protected] Website : www.suryapublications.in Registered Office : New No. 59, 4th Avenue, Opp. to Govt. Boys Hr. Sec. School, Ashok Nagar, Chennai - 600 083. Phone : 044 - 2474 4484 Mobile : 94421 58484 / 94425 58484 Email : [email protected] ENGLISH Govt. Model Question Paper 2019 GANGA's Model Question Papers All 1 & 2 Marks - Key Answers 1+10

ENGLISH · him to send out a cruel proclamation. The entire province was given strict orders to immediately put to death all aged people. Those were barbarous days, and the custom

  • Upload
    others

  • View
    1

  • Download
    0

Embed Size (px)

Citation preview

Model Question Papers(with answers)

Price : ` 40/-

SRI GANGA PUBLICATIONS(A UNIT OF SHYAMALA GROUP)

Corporate Office :No. 1, Sugar Mill Colony,

Salai Kumaran illam,Madurai Road,

Tirunelveli - 627001.Phone : 0462 - 233 8899 / 233 8484Mobile : 94431 58484 / 95978 39822

Email : [email protected]

Website : www.suryapublications.in

Registered Office :New No. 59, 4th Avenue,Opp. to Govt. Boys Hr. Sec. School,Ashok Nagar,Chennai - 600 083.Phone : 044 - 2474 4484Mobile : 94421 58484 / 94425 58484Email : [email protected]

ENGLISH

Govt. Model Question Paper 2019GANGA's Model Question PapersAll 1 & 2 Marks - Key Answers

1+10

Account Name : SURYA PUBLICATIONSAccount Number : 446971431Bank Name : INDIAN BANKIFSC Code : IDIB000T034Branch Name : TIRUNELVELI JUNCTION

Account Name : SRI GANGA PUBLICATIONAccount Number : 928507483Bank Name : INDIAN BANKIFSC Code : IDIB000A031Branch Name : ASHOK NAGAR

Account Name : SURYA PUBLICATIONSAccount Number : 510909010051752Bank Name : CITY UNION BANKIFSC Code : CIUB0000230Branch Name : PALAYAMKOTTAI

Account Name : SRI GANGA PUBLICATIONAccount Number : 512020010022514Bank Name : CITY UNION BANKIFSC Code : CIUB0000230Branch Name : PALAYAMKOTTAI

OUR BANK DETAILS

CHENNAI ACCOUNT

TIRUNELVELI ACCOUNT

10TH STANDARDMODEL QUESTION PAPERS

GOVT. MODEL QUESTION PAPER

X - STANDARD

ENGLISHTime: 3.00 hrs Max. Marks: 100

Instructions:

1) Check the question paper for fairness of printing. If there is any lack of fairness inform the Hall supervisor immediately.

2) Use Blue or Black ink to write or underline.

PART-I

i. Answer all the questions. 14x1=14 ii. Choose the most suitable answer and write the code with

corresponding answer.

Choose the appropriate synonyms for the italicised words. 1. The mother seagull swooped upwards.

a) leap b) rush c) move very quickly d) ascend

2. The attic has always been favourite with children. a) loft b) terrace c) apartment d) strong room

3. It is a 55-foot sailing vessel built indigenously in India. a) fully b) collectively c) innately d) specially

Choose the appropriate antonym for the italicised words. 4. She screamed back mockingly.

a) disrespectfully b) ridiculouslyc) jeeringly d) respectfully

X ENGLISH2 IDEAL

5. We don’t have to use any means of repulsion.

a) attraction b) distaste c) hate d) horror

6. I indulged in banking,

a) took part b) participated c) abstained d) yielded

7. Choose the correct plural form of ‘alga’ from the following: a) algum b) algi c) algae d) algas

8. Formaderivativebyaddingtherightsuffixtotheword–‘document’. a) – ory b) – ise c) – ation d) – ly

9. Choose the correct expansion of the abbreviation ‘SIM’. a) Subscriber Information Module

b) Subscriber Identification Module

c) Student Identification Module

d) School Identification Module

10. Complete the following sentence with the most appropriate phrasal verb given below: The crew ________ of water and food before they could complete their expedition.

a) ran on b) ran about c) ran in d) ran out

11. Choose the suitable option to pair it with the word ‘watch’ to form a compound word. a) hall b) house c) man d) clock

12. Fill in the blank with the most appropriate preposition given below: Mulan heard this ________ her tent.

a) by b) from c) at d) for

13. Complete the following sentence using the most appropriate tense form of the verb given below: After he ________ his lunch, he went across to the window.

Model Question Papers 3

a) will finish b) finish

c) was finishing d) had finished

14. Choose the most appropriate linker from the given four alternatives. ________ he is ninety years old, he is in the pink of health.

a) When b) Since c) Even though d) Yet

Part–II (10x2=20)Section–I

Answer any THREE of the following questions in a sentence or two. 3x2=6 15. Mention the special features of INSV Tarini. 16. What prompted the seagull to fly finally? 17. What was the daily routine of Mr. Sanyal? 18. What were the various things that tempted Mr. Franz to spend

his day outdoors?

Section–II

Read the following sets of poetic lines and answer any THREE of the following. 3x2=6

19. Let us learn to walk with a smile and a song, No matter if things do sometimes go wrong;

a) What does the poet want everyone to learn?

b) What should we do when things go wrong?

20. She’s a lioness; don’t mess with her. She’ll not spare you if you’re a prankster.

a) How is a woman described here?

b) Who is a prankster?

21. Not a flower could he see. Not a leaf on a tree.

a) Who does ‘he’ refer to?

b) Mention the season when he could not see a flower or a leaf on a tree.

X ENGLISH4 IDEAL

22. Beside the house sits a tree It never grows leaves.

a) What is found near the house?

b) Why does it never grow leaves?

Section–III

Answer any THREE of the following: 3x2= 6 23. Rewrite the following sentence to the other voice.

Please assemble in the ground.

24. Rewrite using indirect speech.

“ Where are we going, sir?” asked the aero-coachman.

25. Punctuate the following sentence.

wherefore said miranda did they not that hour destroy us

26. Transform the following sentence into a simple sentence.

As Catherin is a voracious reader, she buys a lot of books.

27. Rearrange the words in the correct order to make meaningful sentences:

a) he saw / when / in the / platform / the train / he rushed.

b) as / I / healthy / are you / am / as.

Section–IV

Answer the following question: 1x2=2 28. A stranger wants to visit the library. Write the steps to

guide him to reach his destination.

Bank

Anna Road

You and thestranger are here

Big

Stre

et

Hospital

Library

Model Question Papers 5

Part–III (10x5=50)

Section - I

Answer any TWO of the following in utmost 10 lines. 2x5=10 29. Describe the struggles undergone by the young seagull to

overcome its fear of flying. 30. ‘Technology is a boon to the disabled’. Justify.31. How did Watson help his friend to arrest the criminal? 32. ‘Man does change with time’ - What were the various changes

that came about in Aditya?

Section–II

Answer any TWO of the following in utmost 10 lines. 2x5=10 33. How is mystery depicted in the poem, ‘The House on Elms

street’? 34. Compare and contrast the attitude of the ant and the cricket.

35. Read the following stanza and answer the questions given below: The weather is always too hot or cold; Summer and winter alike they scold. Nothing goes right with the folks you meet Down on the gloomy Complaining street.

Questions:i) Pick out the rhyming words from the above lines. ii) Write the rhyme scheme of the given stanza. iii) Identify the figure of speech employed in the fourth line of

the given stanza. iv) Pick out the alliterating words.

36. Paraphrase the following stanza. We can pull and haul and push and lift and drive, We can print and plough and weave and heat and light, We can run and race and swim and fly and dive, We can see and hear and count and read and write!

X ENGLISH6 IDEAL

Section–III

Answer any ONE of the following: 1x5=5 37. Rearrange the following sentences in coherent order.

i) Using his powers, Prospero released the good spirits from large bodies of trees.

ii) Prospero and Miranda came to an island and lived in a cave. iii) He raised a violent storm in the sea to wreck the ship of his

enemies. iv) The King of Naples and Antonio the false brother, repented

the injustice they had done to Prospero. v) He ordered Ariel to torment the inmates of the ship.

38. Read the following passage and answer the questions that follow.

The country Shining was governed by a despotic leader who though a warrior, had a great and cowardly shrinking from anything suggestive of failing health and strength. This caused him to send out a cruel proclamation. The entire province was given strict orders to immediately put to death all aged people. Those were barbarous days, and the custom of abandoning old people to die was not in common. The poor farmer loved his aged mother with tender reverence, and the order filled his heart with sorrow. But no one ever thought twice about obeying the mandate of the governor, so with many deep and hopeless sighs, the youth prepared for what at that time was considered the kindest mode of death.

Questions: i) Who governed Shining? ii) What was the cowardly act of the governor? iii) What proclamation did the governor send out? iv) How did the poor farmer treat his mother? v) Did the people obey the governor’s order?

Model Question Papers 7

Section–IV

Answer any FOUR of the following: 4x5=20

39. Prepare an attractive advertisement using the hints given below.

Home appliances – Aadi Sale – 20-50% – Special Combo Offers – Aadhi & Co., Raja Street, Chennai.

40. Write a letter to the manager of a famous daily, ordering subscription for your school library.

41. You are Adhira / Athiran, the school pupil leader of GHSS, Trichy. Prepare a notice on behalf of your school inviting the grandparents of the students to celebrate World Elders’ Day in your school auditorium on the 20th of next month.

42. Look at the following picture and express your views on itinaboutfivesentences.

43. Make notes or write a summary of the following passage. There are many different kinds of books that are published

each year. These are the new titles available for us to read. Besides these, there are books that have been published through the years. Together, there are millions of books available throughout the world in as many languages as are spoken by people. There are different genres in which books are published. There are fiction and non-fiction categories in books, and each

X ENGLISH8 IDEAL

of these categories has many different genres of books. The academic books we study at school belong to the text book category. We study them to complete our syllabus and pass the examinations at the end of each academic session. There are other books that we read for our pleasure and enrichment. We read story books of different types. There are comedy, horror, detective and thriller stories in prose, plays and poetry forms. Books are our best friends.

44. Identify and correct the errors in the following sentences. a) You may speak politely to the elders. b) This is the boy whom won the race. c) He come late to school every day. d) Though he was hungry but he did not eat. e) Is this a book that you wanted to buy?

Section–V

45. Quote from memory. 1×5=5

Let me live .......... back in fear.

Part–IV

46. Write a paragraph of about 150 words by developing the following hints. 2x8=16

a) Many years ago – China – the emperor ordered – one man from – family – join army – Mulan heard – told father she join army – father objected – she is a girl – Mulan – wore – father’s robes cuts her hair – convinced father – she has learnt – Kung fu – no one will find – she is a girl. Mulan left – village – fought bravely – war – given top – position very soon – fever swept – the army – Mulan – sick – doctor examines – finds the truth – spreads the news in the army – everyone objects – to follow a girl leader – Mulan stood tall – gave command – soldiers – followed her attacked enemies – won the battle – Emperor glad – offered Mulan positions – court – Mulan refused – went back village – royal – gifts.

Model Question Papers 9

(OR)

b) Holland – land – below sea level – dikes protected the country – everyone did best to-protect – Holland-Years ago – little – boy Peter – lived Holland – His father – attended – dyke gates – opened – closed dykes – one day – Peter mother – gave cakes to Peter – to be given – old blind friend of Peter – across the dyke – Peter happily left home – Peter visited – old man – returned near by the dyke – heard-water trickling – stopped to see – small hole dyke- called for help – in vain – he put his little – finger – throughout the night – slept near the dyke – morning – found by passer by – alerted the people – Peter and Holland – saved.

47. Read the following paragraph and answer the questions given below:

Kung Fu – ‘kung’ meaning ‘energy’ and ‘fu’ meaning ‘time’ – is a Chinese martial art whose recorded history dates back to around 525 CE, during the Liang dynasty. The man credited with introducing martial arts to China is said to be an Indian monk known as Bodhidarma.

Many people have a misconception that Chinese Kung Fu is about fighting and killing. It is actually based on Chinese philosophy and is about improving wisdom and intelligence. Taoist philosophy is deeply rooted in and had a profound influence on the culture of Chinese martial arts.

The five traditional animal styles of Shaolin Kung Fu are the dragon, the snake, the tiger, the leopard and the crane. The union of the five animal forms clearly displayed the efficacy of both hard and soft movements, of both internal and external energy - this form of Chinese martial arts was known as Shaolin Kung Fu, named after the temple in which it was developed.

X ENGLISH10 IDEAL

Questions: a) Which country does the martial art Kung Fu belong to? b) What is the meaning of the term “Kung Fu”? c) Write any two martial arts of India. d) What are the five animal styles followed in Shaolin Kung Fu?

(OR)

Read the following poem and answer the questions given below: It you can’t be a pine on the top of the hill, Be a scrub in the valley - but be The best little scrub by the side of the rill; Be a bush, if you can’t be a tree. If you can’t be a bush, be a bit of the grass, And some highway happier make; If you can’t be a muskie, then just be a bass- But the liveliest bass in the lake! We can’t all be captains, we’ve got to be crew, There’s something for all of us here. There’s big work to do and there’s lesser to do And the task we must do is the near. If you can’t be a highway, then just be a trail, If you can’t be the sun, be a star; It isn’t by size that you win or you fail- Be the best of whatever you are!

Questions:

a) Where does the best scrub grow?

b) What makes a highway traveler happy?

c) Does size matter? Give reason.

d) What is the underlying theme of the poem?

*****

Model Question Papers 11

Key Answers

GOVT. MODEL QUESTION PAPER

1. c 2. a 3. c 4. d 5. a 6. c 7. c 8. c9. b 10. d 11. c 12. b 13. d 14. c

23. You are requested to assemble in the ground.24. The aero-coachman asked where he was going.25. ‘‘Wherefore”, said Miranda, “did they not that hour destroy us?”26. Being a voracious reader, Catherin buys a lot of books.27. a) When he rushed, he saw the train in the platform. b) I am as healthy as you are.28. (i) Go straight and turn left. (ii) Keep walking along Gandhi Salai. (iii) Walk across at the signal. (iv) You can see the Public Hall on left.44. a) You ought to speak politely. b) This is the boy who won the race. c) He comes late to school everyday. d) Delete ‘Though’ or ‘but’. e) Is this the book you wanted to buy?47. a) China. b) Energy time. c) Silambattam, wrestling. d) dragon, snake, tiger, leopard and crane.

(OR) a) in the valley. b) a bit of the grass. c) No. Can should do one’s best. d) Be the best of whatever you are!

*****

X ENGLISH12 IDEAL

MODEL QUESTION PAPER - 1

X - STANDARD

ENGLISHTime: 3.00 hrs Max. Marks: 100

Instructions:

1) Check the question paper for fairness of printing. If there is any lack of fairness inform the Hall supervisor immediately.

2) Use Blue or Black ink to write or underline.

PART–I

i. Answer all the questions. 14x1=14 ii. Choose the most suitable answer and write the code

with corresponding answer.

Choose the appropriate synonyms for the italicised words. 1. We could hear the tromping of the police.

a) walking slowly b) walking heavilyc) walking quickly d) limping

2. INSV stands for Indian Naval Ship Vessel. a) ranges b) refers c) denotes d) supports

3. M. Hamel was sitting motionless in his chair. a) moved b) immovablec) shapeless d) unenthusiastic

Choose the appropriate antonym for the italicised words. 4. The cops were reluctant to leave.

a) unwilling b) readyc) willing d) indifferent

5. It is a boon to citizens with special needs. a) ordinary b) extraordinaryc) usual d) normal

Model Question Papers 13

6. The butler appeared at the doorway,

a) arose b) emerged c) vanished d) seemed

7. Choose the correct plural form of basis from the following:

a) basises b) basics c) bases d) base

8. Form a derivative by adding the right suffix to the word – custom.

a) – ary b) – ise c) – ation d) – ly

9. Choose the correct expansion of the abbreviation ‘NOTA’.

a) Nun of the Above b) Not of the Above

c) Nothing of the Above d) None of the Above

10. Complete the following sentence with the most appropriate phrasal verb given below:

Students should know how to ________ with their friends.

a) get on b) get about c) get in d) get out

11. Choose the suitable option to pair it with the word ‘mark’ to form a compound word.

a) hall b) house c) man d) book

12. Fill in the blank with the most appropriate preposition given below:

The seagull was scared ________ flying.

a) by b) from c) of d) for

13. Complete the following sentence using the most appropriate tense form of the verb given below:

The milk man ________ (knock) at the door at 5 a.m. every morning.

a) is knocking b) knocked c) was knocking d) knocks

14. Choose the most appropriate linker from the given four alternatives.

________ the cat is away the mice are at play.

a) When b) Since c) Even though d) Yet

X ENGLISH14 IDEAL

Part–II (10x2=20)

Section–I

Answer any THREE of the following questions in a sentence or two. 3x2=6 15. What were the various sounds the brothers heard when they

went downstairs? 16. What sorts of training did the crew undergo before their

expedition? 17. Expand Kim. 18. Why did Holmes want Smith to treat him?

Section–II

Read the following sets of poetic lines and answer any THREE of the following. 3x2=6 19. My heart will keep the courage of the quest, And hope the road’s last turn will be the best

a) What kind of quest does the poet seek here? b) What is the hope of the poet?

20. She’s today’s woman / Today’s woman, dear Love her, respect her, keep her near

a) How should a woman be treated? b) What is the message conveyed by the poetess through the

above lines?

21. We were cast and wrought and hammered to design We were cut and filed and tooled and gauged to fit

a) Who is the speaker here? b) How are the machines designed?

22. What happened inside that house? I really don’t know I guess it will always be a mystery

a) Does the poet know what happened in the house? b) What is the mystery about the house?

Model Question Papers 15

Section–III

Answer any THREE of the following: 3x2= 6 23. Rewrite the following sentence to the other voice.

Premalatha got an award from the Minister of Education.

24. Rewrite using indirect speech. Raga said, “ I am very tired today. I want to have complete rest”

25. Punctuate the following sentence. radhika said to her friend meena i need your help

26. Transform the following sentence into a complex sentence. Due to incessant rain the collector declared schools and colleges a holiday.

27. Rearrange the words in the correct order to make meaningful sentences: a) to switch / him / off / in both / I asked / the lights.b) I / in Gandhiji’s village / had lived / as his house guest /

Section–IV

Answer the following question: 1x2=2 28. Guide to stranger to Malar Hospital. Write the instructions

required.

MALAR Hospital

SBI Bank

Main Road

Bus Stop You

are Here

School

X ENGLISH16 IDEAL

Part–III (10x5=50)

Section–I

Answer any TWO of the following in utmost 10 lines. 2x5=10 29. Narrate the extensive search operation made by the policeman

in the house. 30. Write in detail about the selection and training process which

the crew underwent. 31. Give an account of the last day of M. Hamel in school. 32. How did Holmes trap Mr. Culvert on Smith to confess the

murder?

Section–II

Answer any TWO of the following in utmost 10 lines. 2x5=10 33. Critically analyses the poem “The Secret of the Machines”. 34. How did Henry Van Dyke depict the journey of life?

35. Read the following stanza and answer the questions given below: Let me but live my life from year to year,With forward face and unreluctant soul;Not hurrying to, nor turning from the goal;Not mourning for the things that disappear

Questions:i) Identify the rhyme scheme of the given lines.ii) Identify the rhyming words of the given lines.iii) Pick out the alliterated words in these lines.iv) What literary device is used here?

36. Paraphrase the following stanza. The summer of life she’s ready to see in springShe says, “Spring will come again, my dearLet me care for the one’s who are nearShe’s the woman – she has no fear

Model Question Papers 17

Section–III

Answer any ONE of the following: 1x5=5 37. Rearrange the following sentences in coherent order.

i) The emperor ordered that each one from the family must participate in war.

ii) Of course that’s true”, said Mulan.iii) She poured her father a cup of tea and handed it to him.iv) But I have an idea.v) I can’t send your brother since he is too little.

38. Read the following passage and answer the questions that follow.

With the help of these spirits, Prospero could command the winds, and the waves of the sea. By his orders they raised a violent storm, in the midst of which, he showed his daughter a fine large ship, which he told her was full of living beings like themselves. “Oh my dear father,” said she, “if by your art you have raised this dreadful storm, have pity on their sad distress. See the vessel will be dashed to pieces. Poor souls! they will all perish.” “Be not so amazed, daughter Miranda,” said Prospero; “there is no harm done. I have so ordered it, that no person in the ship shall receive any hurt. What I have done has been in care of you, my dear child. You are ignorant. Can you remember a time before you came to this cell? I think you cannot, for you were not then three years of age”.

Questions:a) What did Prospero do with the help of the good spirits?b) Whom did Miranda take pity on?c) What did Prospero assure?d) How old was Miranda when she came to the island?e) Who were in the large ship?

X ENGLISH18 IDEAL

Section–IV

Answer any FOUR of the following: 4x5=20 39. Prepare an attractive advertisement using the hints

given below. Pen world – mightier than the sword – variety of pens – different dazzling colours – writes smoothly – prices from Rs.10 to Rs. 1000/- for gifts for personal use.

40. Write a letter to the Editor of a newspaper complaining about the nuisance of loud speakers in your locality.

41. You are Ganesh / Gayathiri, Head boy / Head Girl of your school. Write a notice for your school notice board informing the students about the “Fancy Fete” that is going to be organized in your school campus on the 25th of this month.

42. Look at the following picture and express your views on itinaboutfivesentences.

43. Make notes or write a summary of the following passage. Guyana is a small country situated in the northern coast of

South America, on the Atlantic Ocean. People of Indian ancestry form nearly 50% of its population, the rest are local Amerindians, and people of European and Chinese descent.

Christopher Columbus is believed to have discovered. Guyana in 1498. The Dutch landed there in the late 16th

century and the local Amerindians welcomed them as trading

Model Question Papers 19

partners. But the Dutch became permanent settlers and began ruling the country. Britain took over the country in 1796, and ceded the country with Great Britain and named it British Guyana in 1831. In 1834, slavery was abolished, and thousands of indentured labourers were brought from India, Portugal and China to replace the slaves working in sugarcane plantations. Two ships from Calcutta carried the Indians.

The Whitby sailed with 249 people on 13th January 1838 and arrived in Guyana on 5th May. The Hesperus left 6 days later with 165 passengers and arrived in Guyana late on the night of 5th May. Today the country’s population is nearly 8,00,000 in an area of 214969 sq.km. The average population density for Guyana is less than four persons per sq.km.

44. Identify and correct the errors in the following sentences. a) Despite being old and he is active. b) Abi is inferior than Raga in games. c) Ram is one of the best student present.d) Your counsel will benefit to me. e) Raju met in an accident.

Section–V

45. Quote from memory. 1×5=5They too, aware .......... different from our own.

Part–IV

46. Write a paragraph of about 150 words by developing the followinghints. (2x8=16)a) Prospero – Duke of Milan – art of magic – brother Antonio

– overthrow Prospero – King of Naples – Prospero and his daughter – twelve years – released good spirits from Sycorax – creates storms – Ariel helps the enemies – Ferdinand arrives – Miranda and Ferdinand – attractive – Prospero tests – Ferdinand – realizes his mistakes – repented – Prospero – restored the kingdom.

X ENGLISH20 IDEAL

(OR)

b) Uncle Philip – inventor – mechanical engineer – bachelor – lived lonely life – mania for collections of stones – he called Tom – told a strange story – Tom as heir – to pay for his funeral – Leaving an iron box – with precious stones – not to shake it – dried after a week – Tom found a letter – box contained a lot of rubies, diamonds, pearls – also dynamite – if opened with doubt – explodes – possible ways to open – consulted Professor Clinch – but no solution – read about explosives – led to suspicion – changed his name and occupation – finally – handed over the box – Preservation of Human Vivisection.

47. Read the following paragraph and answer the questions given below:

The Vatican is the world’s smallest country which became independent in 1929. It occupies 109 acres in the Italian capital Rome. It has been the spiritual and administrative centre of the Roman Catholic Church. At the heart of the Vatican’s St. Peter’s Basilica, the world’s largest Christian Church. Inside the Vatican palace are the living quarters of the Pope and the offices where the officials work. The palace includes chapels, a library and museums. Vatican city state has its own small ‘army’ of soldiers called the Swiss Guard, who maintain order and protect the Pope. About 1000 people live here. All the people here are Christians and they speak Italian.

Questions:

a) What is the specialty of the Vatican?

b) Describe the location of the Vatican city.

c) What is St Peter’s Basilica? Where is it?

d) What can one see in the palace?

Model Question Papers 21

(OR)

Read the following poem and answer the questions given below: When I behold the heavens as in their prime,And then the earth, though old, still clad in green,The stones and trees insensible of time,Nor age nor wrinkle on their front are seen;If winter come, and greenness then doth fade,A spring returns, and they’re more youthful made.But man grows old, lies down, remains where once he’s laid

By birth more noble than those creatures all,Yet seems by nature and by custom cursed —No sooner born but grief and care make fallThat state obliterate he had at first;Nor youth, nor strength, nor wisdom spring again,Nor habitations long their names retain,But in oblivion to the final day remain.

Shall I then praise the heavens, the trees, the earth,Because their beauty and their strength last longer?Shall I wish there or never he had birth,Because they’re bigger and their bodies stronger?Nay, they shall darken, perish, fade, and die,And when unmade so ever shall they lie;But man was made for endless immortality.

Questions:

a) How is the old earth dressed up?

b) What happens when winter comes?

c) How is human birth described?

d) What will happen to the strength?*****

X ENGLISH22 IDEAL

Key Answers

MODEL QUESTION PAPER - 1

1. c 2. c 3. b 4. c 5. a 6. c 7. c 8. a9. d 10. a 11. d 12. c 13. d 14. a

23. An award was got by Premalatha from the Minister of Education.24. Raga said that she was very tried that day and she wanted to have

complete rest.25. Radhika said to her friend, “Meena, I need your help”.26. As it rained incessantly, the collector declared schools and

colleges a holiday.27. a) I asked him to switch off the lights. b) I had lived in Gandhiji’s village as his house guest.28. (i) Walk straight and turn left. (ii) Keep walking till you see a lane to your right. (iii) Walk across the main road. (iv) You will see Malar Hospital on left.44. a) Delete ‘and’ b) superior to c) students d) Delete ‘to’ e) met with an accident.47. a) the world’s smallest country. b) It occupies 109 acres in Rome.- c) Chirstain Church - Vatican. d) Chapels, a library and museums.

(OR) a) in green. b) greenness fades away. c) By birth, he is more noble but cursed by nature and custom. d) Strength fades away.

*****

Model Question Papers 23

MODEL QUESTION PAPER - 2

X - STANDARD

ENGLISHTime: 3.00 hrs Max. Marks: 100

Instructions:

1) Check the question paper for fairness of printing. If there is any lack of fairness inform the Hall supervisor immediately.

2) Use Blue or Black ink to write or underline.

PART–I

i. Answer all the questions. 14x1=14 ii. Choose the most suitable answer and write the code

with corresponding answer.

Choose the appropriate synonyms for the italicised words. 1. The sight of the food maddened him.

a) furious b) composed c) pacified d) specified

2. Trying to revive old childhood memories may prove disappointing. a) bring up b) bring back c) bring down d) bring out

3. It was the gaunt face. a) flourishing b) freshc) skeletal and thin d) refreshing

Choose the appropriate antonym for the italicised words. 4. ________ the night had been distinctly a defeat for them.

a) clearly b) vaguely c) dimly d) brightly

5. The owner of the shop. Now over sixty, was rustic in appearance. a) urbane b) rural c) village d) country

6. it is a boon to citizens with special needs. a) bounty b) curse c) bane d) gifted

X ENGLISH24 IDEAL

7. Choose the correct plural form of bacterium from the following: a) bacteriums b) bacteria c) bacteriea d) bacterium

8. Formaderivativebyaddingtherightsuffixtotheword–‘prepare’.a) – ory b) – ise c) – ation d) – ly

9. Choose the correct expansion of the abbreviation ‘NEET’. a) National Exam Entrance Testb) National Education Entrance Test c) National Evaluation Educational Test d) National Eligibility Entrance Test

10. Complete the following sentence with the most appropriate phrasal verb given below: The Board of Directors ________.a) fell on b) fell about c) fell in d) fell apart

11. Choose the suitable option to pair it with the word ‘time’ to form a compound word. a) hall b) house c) man d) piece

12. Fill in the blank with the most appropriate preposition given below: The owner of the shop looked ________ the fence. a) up b) down c) around d) in

13. Complete the following sentence using the most appropriate tense form of the verb given below: Rare plants ________ (find) in the silent valley. a) is found b) foundc) are found d) has been found

14. Choose the most appropriate linker from the given four alternatives. Hari is rich ________ he helps others a lot. a) When b) Since c) Even though d) Yet

Model Question Papers 25

Part–II (10x2=20)

Section–I

Answer any THREE of the following questions in a sentence or two. 3x2=6 15. What was the daily routine of the Sanyal? 16. How many people in India suffer with disability? 17. M. Hamel was gazing at many things. What were they? 18. Why did Holmes plead with Smith?

Section–II

Read the following sets of poetic lines and answer any THREE of the following. 3x2=6

19. There’s a family nobody likes to meet They live, it is said, on Complaining Street

a) Where does the family live?

b) Why do you think the street is named as “Complaining Street?

20. A silly young cricket accustomed to sing Through the warm, sunny months of gay summer and spring

a) What are the seasons mentioned here?

b) What was the routine of the cricket?

21. We can pull and haul and push and lift and drive We can print and plough and weave and heat and light

a) What do you mean by pull and haul?

b) Are machines used for farming?

22. They, too, aware of the sun and air and water Are fed by peaceful harvests, by war’s long winter starv’d

a) What is common for all of us?

b) Are machines used for farming?

X ENGLISH26 IDEAL

Section–III

Answer any THREE of the following. 3x2= 6

23. Rewrite the following sentence to the other voice. The manager appointed many office assistants.

24. Rewrite using direct speech. The Captain of the Indian team assured that they would win the series.

25. Punctuate the following sentence. my grandfather said ram don’t worry everything will be alright.

26. Transform the following sentence into a simple sentence. Tell me what the time is.

27. Rearrange the words in the correct order to make meaningful sentences: a) headed / Kapil Dev / the winning / cricket / Indian / team /

was / by.b) loan / apply / you / if / for / you / a / get / will / immediately

/ it.

Section–IV

Answer the following question: 1x2=2

28. Guide the stranger to the State Bank of India. Write three instructions by way of helping him.

MAIN ROAD

Raj Stores

State Bank

of India

Bus Stop You are

Here

Model Question Papers 27

Part–III 10x5=50

Section–I

Answer any TWO of the following in utmost 10 lines. 2x5=10 29. Describe the funny incident that caused the confusion in the

house. 30. Highlight the factors responsible for all – women Indian Navy

Crew to carry out their expedition. 31. ”Technology is the boon to the disabled” Justiify. 32. How did Holmes trap Mr. Culverton Smith to confess the murder?

Section–II

Answer any TWO of the following in utmost 10 lines. 2x5=10 33. Describe the journey of life as depicted in the poem by Henry

Van Dyke. 34. How is the mystery depicted in the poem?

35. Read the following stanza and answer the questions given below: They growl at the rain and they growl at the sun,.In fact, their growling is never done.And if everything pleased them, there isn’t a doubtThey’d growl that they’d nothing to grumble about

i) Mention the figure of speech used in the first line.ii) What is the rhyme scheme used here?iii) What are the rhyming words?iv) Pick out the words which are alliterated in the last line.

36. Paraphrase the following stanza. A woman is beauty innate A symbol of power and strengthShe puts her life at stakeShe’s real, she’s not fake!

X ENGLISH28 IDEAL

Section–III

Answer any ONE of the following: 1x5=5 37. Rearrange the following sentences in coherent order.

– Francis Bennet went on into the reporter’s straw.

– That morning Francis Bennet awoke in a rather bad temper.

– The telephone, completed by the telephote, is another of our time’s conquests

– He quickly jumped out of the bed and went into his mechanized dressing room.

– “Well,.Cash, what have you got?”

38. Read the following passage and answer the questions that follow.

‘You’re both quite mistaken’, their father hastened to explain, seeing his wife’s horrified expression. ‘Zigzag is a most harmless, unusual and lovable bird. ‘Great hit indeed!’ Mrs.Krishnan didn’t bother to hide her sarcasm, and continued, ‘Considering that the boomerang sliced through all the TV aerials in the neighbourhood, caused permanent damage to several cars in the parking lot, and knocked out our watchman cold, with the force you threw it.’ ‘But Zigzag is different. Somu says we are sure to love Zigzag,’ soothed Dr. Krishnan, ‘because the bird can talk and sing in about twenty-one different language – mostly African languages, of course. When it sings, it moves the listeners to tears. ‘This is Zigzag! Announced Visu with a flourish. ‘His full name is Ziggy-Zagga-King-of-the-Tonga. How I’m going to miss him! So beautifully he talks! He can even recite French Poetry!’

Questions:i) Who is Zigzag?ii) What commotion did the boomerang cause in the

neighbourhood?

Model Question Papers 29

iii) What are the specialities of Zigzag?iv) Who recites French poetry?v) What is the full name of Zigzag?

Section–IV

Answer any FOUR of the following. 4x5=20

39. Prepare an attractive advertisement using the hints given below.

Singapore mobiles – smart phones – accessories – SIM cards – Recharge – Free Power banks on Mobile purchase – No. 1, Toll Gate, Vellore.

40. Write a letter to your friend inviting him to spend summer vacation with you.

41. You are the School Pupil Leader of VVV Matriculation Hr.Sec.School, Dharmapuri. Your school principal has requested you to inform the students about a trip to Yercaud for 3 days. Prepare a notice giving the details such as date of journey, mode of transportation, amount, dress code etc.

42. Look at the following picture and express your views on itinaboutfivesentences.

X ENGLISH30 IDEAL

43. Make notes or write a summary of the following passage: Trees not only supply us with many of the conveniences of

our daily life, they do much more than that. They support the life of living things. They help to replace the oxygen in the air constantly being used up and turned into carbon-dioxide when animals breathe and things burn. The green leaves of trees absorb the carbon-dioxide from the air and with the help of sunlight break it up into carbon and oxygen. The carbon is used to make starch and oxygen is released into the atmosphere. That is how trees help to replace the oxygen in the atmosphere. The green cells of leaves are wonderful little laboratories, where all the starch in the world is produced. Since starch forms an important part of the food of men and animals, their life depends on the work done by the green cells of plants. Thus trees are great friends of man.

44. Identify and correct the errors in the following sentences. a) Though Prema likes coffee but she prefers tea. b) One of the girl look happy.. c) Children bursted into peals of laughter. d) My uncle bought many furntiures. e) Children prefer ice cream than coffee.

Section–V

45. Quote from memory. 1×5=5The summer of life .......... she has no fear.

Part–IV

46. Write a paragraph of about 150 words by developing the followinghints. (2x8=16)a) Dr. Krishnan’s family – Vishu and the pet bird – Dr. Somu

requested – take care – harmless bird – Zigzag – deposited – blades of fan – slept – snored loudly – spoiled Mrs. Krishnan’s painting – took to his clinic – changed – wanted to keep for ever.

Model Question Papers 31

(OR)b) Story set in 2889 – Francis Bennet – director – Earth Herald –

leading newspaper – phonotelephote – mechanized dressing room – astronomical news – advertisements on clouds – lunch via pneumatic tubes – travel to Niagara – an aero car – inventors.

47. Read the following paragraph and answer the questions given below:

Preserving Nature is man’s prime responsibility. The earth does not belong to us for us to use and abuse. It belongs to our children. As man became civilized, he slowly lost his attachment with Nature. In his quest for development, man cleared pristine forests and polluted the environment. The magnitude of destruction increased in proportion to the magnitude of his greed. As man drifted away from Nature, his problems increased and his heart became hard and cold. The many calamities, misfortunes and problems of our times are but the manifestation of this greed. This is why we have injured people, animals and environment. If we want to set things right we must reconnect with Nature.

Questions: a) What is the prime responsibility of man?b) Whom does the earth belong to?c) When does man lose attachment with Nature?d) What is the manifestation of man’s greed?

(OR)

Read the following poem and answer the questions given below: And now there came both mist and snow, And it grew wondrous cold: And ice, mast-high, came floating by,As green as emerald.

X ENGLISH32 IDEAL

And through the drifts the snowy cliffsDid send a dismal sheen: Nor shapes of men nor beasts we ken – The ice was all between.

The ice was here, the ice was there, The ice was all around: It cracked and growled, and roared and howled, Like noises in a sound! ,

At length did cross an Albatross, Thorough the fog it came; As if it had been a Christian soul, We hailed it in God ‘s name.

Questions:

a) According to the poet what came there?

b) How did the ice look like?

c) Where could the poet see the ice?

d) Which crossed through the fog?

*****

Key Answers

MODEL QUESTION PAPER - 2

1. a 2. b 3. c 4. b 5. a 6. b 7. b 8. c9. d 10. d 11. d 12. c 13. c 14. d

23. Many office assistants were appointed by the manager.24. The captian of the Indian team said,”You will win the series”.25. My grandfather said, “Ram, don’t worry. Everything will be

alright”.26. Tell me the time.

Model Question Papers 33

27. a) The winning Indian cricket team was headed by Kapil Dev. b) If you apply for a loan you will get it immediately.28. (i) Go straight and turn left. (ii) Keep walking along the main road. (iii) Take the lane to your left. (iv) After walking some distance, you can find the SBI on your

right.44. a) Delete ‘though’ or ‘but’ b) One of the girls looks happy. c) Children burst into peals of laughter. d) My uncle bought many furniture. e) Children prefer ice-cream to coffee.47. a) Preserving nature. b) It belongs to our children. c) When man became civilized, he lost attachment with nature. d) the magnitude of destruction.

(OR) a) Mist and snow. b) as green as emerald. c) The ice was all around. d) an Albatross.

*****

X ENGLISH34 IDEAL

MODEL QUESTION PAPER - 3

X - STANDARD

ENGLISHTime: 3.00 hrs Max. Marks: 100

Instructions:

1) Check the question paper for fairness of printing. If there is any lack of fairness inform the Hall supervisor immediately.

2) Use Blue or Black ink to write or underline.

PART –I

i. Answer all the questions. 14x1=14 ii. Choose the most suitable answer and write the code with

corresponding answer.

Choose the appropriate synonyms for the italicised words.

1. The great expanse of sea stretched down beneath. a) large / vast space b) narrow spacec) small space d) deep space

2. He said in a hopeless tone of a despondent beagle. a) angry b) affluentc) despairing d) strong

3. They were apprehensive and supportive too.a) confident b) inquisitivec) anxious d) special

Choose the appropriate antonym for the italicised words.

4. I had counted on the commotion to get to my desk without being seen. a) confusion b) disturbance c) unrest d) calmness

5. ”How ignorant you are! Watson!” He said with a groan. a) shout b) growl c) laugh d) cry

Model Question Papers 35

6. So we can now look forward to a more inclusive way of learning. a) comprehensive b) exclusivec) totally d) overall

7. Choose the correct plural form of ‘staff’ from the following: a) staffs b) staffes c) staff d) staves

8. Formaderivativebyaddingtherightsuffixtotheword– ‘real’. a) – ary b) – ise c) – ation d) – ly

9. Choose the correct expansion of the abbreviation ‘GPS’. a) Geometric Position Systemb) Global Pointing Structure c) Globe Positioning Systemd) Global Positioning System

10. Complete the following sentence with the most appropriate phrasal verb given below: The officer ________ their demands.a) looked up b) looked into c) looked for d) looked at

11. Choose the suitable option to pair it with the word ‘auction’ to form a compound word. a) hall b) house c) man d) piece

12. Fill in the blank with the most appropriate preposition given below: He traveled ________ the world in eighty days.a) up b) down c) around d) in

13. Complete the following sentence using the most appropriate tense form of the verb given below: Water ________ (boil) at O Centigrade. a) is boiled b) boilsc) are boiled d) has been boiled

X ENGLISH36 IDEAL

14. Choose the most appropriate linker from the given four alternatives. The orator’s fluency is good ________ his speech is monotonous.a) when b) since c) even though d) but

Part–II (10x2=20)

Section–I

Answer any THREE of the following questions in a sentence or two. 3x2=6 15. How did the bird try to reach its parents without having to fly? 16. Name a few Indian innovations which are helpful to the disabled

and make their day to day life easier. 17. Why was the narrator sorry to have paid attention to the

footsteps? 18. What was the condition of the attic?

Section–II

Read the following sets of poetic lines and answer any THREE of the following. 3x2=6 19. Though our smoke may hide the Heavens from your eyes It will vanish and the stars will shine again

a) What will vanish? b) Explain the phrase ‘the stars will shine again’

20. Not hurrying to, nor turning from the goal Not mourning for the things that disappear

a) Why do you think the poet is not in a hurry? b) What should one not mourn for?

21. Let us learn to walk with a smile and a song No matter if things do sometime go wrong

a) What does the poet expect everyone to learn? b) What should one not mourn for?

22. Don’t ever try to saw her pride She knows how to thaw you, saw you – so beware

Model Question Papers 37

a) What do the words thaw and saw mean here? b) What is the tone of the author?

Section–III

Answer any THREE of the following: 3x2= 6 23. Rewrite the following sentence to the other voice.

You are making the cake now.

24. Rewrite using indirect speech. The Principal said, “ Don’t waste time and concentrate on studies”

25. Punctuate the following sentence. he was near the sea now the flying straight over it facing out over the ocean

26. Transform the following sentence into a simple sentence. Latha is sick but she attends the rehearsal.

27. Rearrange the words in the correct order to make meaningful sentences:a) many Indians / sandalwood / to be / is considered / sacred by.b) in ancient times / sandal and turmeric / blemish free skin /

for a / was used / a paste of / by women.

Section–IV

Answer the following question: 1x2=2 28. You are near the school. An old man asks you to direct

him to the Indian Bank. Guide him with your directions in about 50 words.

Temple

Indian BankBus Stop

Raja StoreYour are here School

MAIN ROAD

X ENGLISH38 IDEAL

Part–III 10x5=50

Section–I

Answer any TWO of the following in utmost 10 lines. 2x5=10 29. Describe the funny incident that caused the confusion in the

house. 30. Write in detail the selection and training processes which the

crew underwent.31. How do we use technology in our day to day lives? 32. Why does M. Hamel say that we must guard our language?

Section–II

Answer any TWO of the following in utmost 10 lines. 2x5=10 33. What is the importance of machines in our life? 34. From the poem “The Grumble Family’ what kind of behavior

does the poet want the readers to possess?

35. Read the following stanza and answer the questions given below: Let me but live my life from year to yearWith forward face and unreluctant soulNot hurrying to, nor turning from the goalNot mourning for the things that disappear

Questions:i) What literary device is used here?ii) Pick out the words of alliteration in the above lines.iii) Write out the rhyming words.iv) Write the rhyme scheme of these lines.

36. Paraphrase the following stanza. She’s a lioness; don’t mess with herShe’ll not spare you if you’re a pranksterDon’t ever try to saw her pride, her self respectShe knows how to thaw you, saw you – so beware

Model Question Papers 39

Section–III

Answer any ONE of the following: 1x5=5 37. Rearrange the following sentences in coherent order.

i) Somu’s old cook Visu brought Zigzag to Dr. Krishnan’s house.ii) Somu told Dr. Krishnan that Zigzag could talk and sing in

about twenty-one languages.iii) But Zigzag transferred all the walnuts and fruits to the

chandelier and on the blades of the ceiling fan.iv) Dr. Krishnan’s cousin Somu asked him to keep his pet bird

Zigzag with him as he left for Alaska.v) Arvind, the doctor’s son gave Zigzag a plate of juicy fruit

slices and some nuts.

38. Read the following passage and answer the questions that follow.

But others called out, in voices just as loud, “With Mulan, we win every battle!” They said, “Stay away from our General!” Just then, a soldier ran up. “Everyone!” he called. “A surprise attack is coming!” Mulan heard this from inside her tent. She got dressed and went outside. She was not yet strong, but stood tall. She told the soldiers where they must go to hide so they could attack when the enemy came. But they must get there fast! The soldiers, even those who did not like that their General was a woman, could tell that Mulan knew what she was talking about. It worked! The battle was won. It was such a big victory that the enemy gave up, at last. The war was over, and China was saved! You can be sure that after that last battle, no one cared anymore that Mulan was a woman. The Emperor was so glad that Mulan had ended the long war, he set aside the rule about being a woman. “Mulan, stay with me in the palace,” he said. “Someone as smart as you would be a fine royal adviser.” Mulan bowed deeply. “You are too kind, Sire,” she said. “But if you please, what I wish most of all is to return home to my family.” “Then at least take these fine gifts,” said the Emperor. “So everyone at your home and village will know how much the

X ENGLISH40 IDEAL

Emperor of China thinks of you.” Mulan returned to her village with six fine horses and six fine swords. Everyone cheered that she was safe. The person who had saved China was their very own Mulan!

Questions:i) Why did The Emperor need troops?ii) What did The Emperor do after the long war ended? iii) Why was Mulan invited to the palace?iv) What made Mulan feel better during hard times?v) How did the Emperor honour Mulan?

Section–IV

Answer any FOUR of the following: 4x5=20 39. Prepare an attractive advertisement using the hints

given below. Footking footwear – shoes, chappals – imported footwear – 20% discount – gents – ladies – kids – stylish shoes – comfortable slippers

40. Write a letter to the head of the Vodafone office enquiring regarding the internet broadband scheme launched recently.

41. You are Ganesh / Gayathiri, Headboy / Headgirl of your school. Write a notice for your school notice board informing the students about a film show that is going to be organized in your school campus on the 15th instant.

42. Look at the following picture and express your vie ws on itinaboutfivesentences.

Model Question Papers 41

43. Make notes or write a summary of the following passage: Trees not only supply us with many of the conveniences of

our daily life, they do much more than that. They support the life of living things. They help to replace the oxygen in the air constantly being used up and turned into carbon-dioxide when animals breathe and things burn. The green leaves of trees absorb the carbon-dioxide from the air and with the help of sunlight break it up into carbon and oxygen. The carbon is used to make starch and oxygen is released into the atmosphere. That is how trees help to replace the oxygen in the atmosphere. The green cells of leaves are wonderful little laboratories, where all the starch in the world is produced. Since starch forms an important part of the food of men and animals, their life depends on the work done by the green cells of plants. Thus trees are great friends of man. Identify and correct the errors in the following sentences.

44. Identify and correct the errors in the following sentences.a) Sitting near the window and he saw the child.b) Nathan is week in English.c) My uncle deals with silver. d) Raj with his friends have gone to market. e) Let’s wait here, shall I?

Section–V

45. Quote from memory. 1×5=5The summer of life .......... she has no fear.

Part–IV

46. Write a paragraph of about 150 words by developing the followinghints. (2x8=16)a) Dr. Krishnan’s family – Vishu and the pet bird – Dr. Somu

requested – take care – harmless bird – Zigzag – deposited – blades of fan – slept – snored loudly – spoiled Mrs. Krishnan’s painting – took to his clinic – changed – wanted to keep for ever.

X ENGLISH42 IDEAL

(OR)b) Story set on 2889 – Francis Bennet - director – Earth Herald –

leading newspaper – phonotelephote – mechanized dressing room – astronomical news – advertisements on clouds – lunch via pneumatic tubes – travel to Niagara – an aero car – inventors.

47. Read the following paragraph and answer the questions given below:

Our earth is the only planet with a variety of plants, animals and the microorganisms living either in the aquatic or the terrestrial habitats. An environment with rich biological diversity is the basis for human existence. The equilibrium status of living and non-living factors in an environment is known as balance in nature. It is disturbed at times due to natural calamities like storm, flood, pests, outbreaks and fire. Rapid population growth, industrialization and urbanization are the causes for deforestation exploiting fauna and flora against ecosystem. Indian Parliament has passed the Wildlife Protection Act in 1972 and the Forest Conservation Act in 1980 to preserve and protect our natural wealth.

Questions: a) How does the earth differ from the other planets? b) What is balance in nature? c) Mention some natural calamities that disturb the balance in

nature. d) Which Act has been passed in our country to protect the

forests and when?

(OR)

Read the following poem and answer the questions given below: In the middle of the cityIs an open space called a Park;

Model Question Papers 43

It is difficult for us to do what we like thereEven after dark.

In the middle of the Park, there is a statue,A huge man made of stone;We are not allowed to climb his legs orScribble on his trousers,

He has to be left alone,In the middle of the grass, there is some waterSurrounded by an asphalt path; We are forbidden to fish or throw stones into itOr swim or take a bath.

Questions:a) What is the open space called?b) What is not possible even after dark?c) Who do you think is forbidden from running freely in the park?d) Whose statue is in the park?

*****

Key Answers

MODEL QUESTION PAPER - 3

1. a 2. c 3. c 4. d 5. c 6. b 7. c 8. b9. d 10. b 11. a 12. c 13. b 14. d

23. The cake is being made by you now.24. The principal asked us not to waste time and to concentrate on

studies.25. He was near the sea now, flying straight over it, facing out over

the ocean.26. In spite of being sick, Latha attends the rehearsal.27. a) Sandalwood is considered sacred by many Indians.

X ENGLISH44 IDEAL

b) In ancient times, the paste of sandal and turmeric was used for a blemish free skin by women.

28. (i) Walk straight and turn left. (ii) Go past the bus stop and you will find a lane on left. (iii) Turn left and keep walking. (iv) You can find the Indian Bank opposite Raja store.44. a) Delete ‘and’. b) weak c) deals in silver. d) has gone to market. e) shall we?47. a) Our earth is the only planet with a variety of plants, animals

and the microorganism living either in the aquatic or the terrastrial habitats.

b) The equilibrium status of living and non-living factors in an environment is known as balance in nature.

c) Natural calamities like storm, flood, pests, outbreaks and fire are some natural calamities that disturb the balance in nature.

d) Wildlife Protection Act in 1972 and the Forest Conservation Act in 1980 were passed in our country to protect the forests.

(OR) a) park. b) It is difficult to do what we like. c) the children. d) a huge man.

*****

Model Question Papers 45

MODEL QUESTION PAPER - 4

X - STANDARD

ENGLISHTime: 3.00 hrs Max. Marks: 100

Instructions:

1) Check the question paper for fairness of printing. If there is any lack of fairness inform the Hall supervisor immediately.

2) Use Blue or Black ink to write or underline.

PART –I

i. Answer all the questions. 14x1=14 ii. Choose the most suitable answer and write the code with

corresponding answer.

Choose the appropriate synonyms for the italicised words.

1. Communicate and control our environment like turning on the lights and out. a) country b) localityc) surrounding d) unfulfillment

2. They were beckoning to him. a) sadly b) invitec) strong d) narrow

3. David will now speak in complete sentences with correct syntax. a) structure b) diagram c) system d) disorder

Choose the appropriate antonyms for the italicised words. 4. The whole school seemed so strange.

a) peculiar b) queer c) crazy d) familiar

5. Technology is also boon to the citizens with special needs. a) curse / bane b) bare c) gift d) moments

X ENGLISH46 IDEAL

6. He screamed with fright and attempted to rise again. a) calm b) energized c) whimper d) cowardice

7. Choose the correct plural form of ‘Chair’ from the following. a) chaires b) chairs c) chairas d) chairaes

8. Form the derivative by adding the right suffix to theword–‘Enjoy’.a) able b) ment c) tion d) sub

9. Choose the correct expansion of the abbreviation ‘ROM’. a) Real Only Memory b) Random Only Memory c) Read Only Memory d) Release Only Monitory

10. Complete the following sentence with the most appropriate phrasal verb given below. I never ________ in the gym. a) work in b) work out c) work away d) work up

11. Choose the suitable option to pair with the word ‘Play’ to form a compound word a) real b) base c) back d) wind

12. Fill in the blank with most appropriate preposition given below: Share the apple ________ your friends. a) from b) above c) among d) until

13. Complete the following sentence using the most appropriate tense form of the given below:The boys usually ________ (play) cricket in this ground. a) is playing b) are playing c) play d) played

14. Choose the most appropriate linker from the given four alternatives.

Model Question Papers 47

________ the bad weather, they decided to have a picnic. a) Although b) In case c) Despite d) Beside

Part–II (10x2=20)

Section–I

Answer any THREE of the following question in a sentence or two. 3x2=6 15. Why did the seagull fail to fly? 16. Name a few Indian innovations which are helpful to the disabled

and make their day to day life earlier.17. What is the future of technology? 18. Why did M. Hamel blame himself?

Section–ll

Read the following sets of poetic lines and answer any THREE of the following. 3x2=6 19. Not hurrying to, nor turning from the goal; Not mourning for the things that disappear

a) Why do you think the poet is not in a hurry? b) What should one not mourn for?

20. And now if you will set us our task We will serve you four and twenty hours a day

a) in the given line open conditional clause is used. Why is the future tense “will set” and “will serve” used in both the clauses?

b) Whose task is referred to as ‘Our task’?

21. They too aware of sun, air and water, Are fed by peaceful harvest, by war’s long winter starv’d

a) What is common for all of us? b) How are we fed?

22. Our hells of fire, and dust outrage the innocence Of air that is everywhere our own.

a) What outrages the innocence?

X ENGLISH48 IDEAL

b) Who are the foreign?

Section–lll 3x2=6

Answer any THREE of the following: 23. Rewrite the following sentence to the other voice.

Radha is playing cricket very well.

24. Rewrite using indirect speech. Shiva said to his friend, “Please give me a hundred rupees.”

25. Punctuate the following sentence. what a trouble must I have been to you then

26. Transform the following sentence into a compound sentence. Walk carefully lest you should fall down.

27. Rearrange the words in the correct order to make meaning ful sentences:a) at the conference / very informative / address / was / your

keynote. b) neck / together / I / will / and / feet / tie / your.

Section–IV

Answer the following question: 1x2=2 28. You need to go to the SBI BANK. How will you go there?

Describe your route.

SBI Bank

Mount Road

Residential Flats

Head Post Office

School Fast FoodCycle

Parking

GANDHI ROAD

1st S

treet

2nd S

treet

Railyway Station

You are Here X

Model Question Papers 49

Part–III (10x5=50)

Section–I

Answer any TWO of the following in about 10 lines. 2x5=10 29. What was the message of the lesson ‘His first flight.’ 30. Technology is a boon to the disabled - Justify. 31. We appreciate the value of something only when we are about

to lose it Explain this with reference to the French language and M.Hamel.

32. Narrate the extensive search operation made by the policeman in the house.

Section–ll

Answer any TWO of the following in about 10 lines. 2x5=10 33. Describe the journey of life as depicted in the poem by Hendry

Van Dyke. 34. The poem “No men are foreign” has a greater relevance in

today’s world. Elucidate.

35. Read the following stanza and answer the question given below: So let the way wind up the hill or down, O’er rough or smooth, the journey will be joy Still seeking what I sought when but a boy, New friendship, high adventure, and a crown,

Questions:a) Pick out the rhyming word. b) Pick out the rhyming scheme. c) What is the figure of speech. d) Pick out the alliterated words.

36. Paraphrase the following stanza. We can pull and haul and push and lift and drive

X ENGLISH50 IDEAL

We can print and plough and weave and heat and lightWe can run and race and swim and fly and diveWe can see and hear and count and read and write

Section–III

Answer any ONE of the following: 1x5=5 37. Rearrange the following sentences in coherent order.

1. Ariel was instructed to bring Ferdinand, the prince of Naples to his cave.

2. He ordered Ariel to torment the inmates of the ship. 3. Using his powers, Prospero released the good spirits from

large bodies of trees. 4. Prospero and Miranda came to an island and lived in a cave. 5. He raised a violent storm in the sea to wreak the ship of his

enemies.

38. Read the following passage and answer the questions that follow.

Bungee jumping is an activity that involves jumping from a tall structure while connected to a long elastic cord. The tall structure is usually a fixed object, such as a building, bridge or crane; but it is also possible to jump from a movable object such as a hot – air – balloon or helicopter, that has the ability to hover above the ground. The thrill comes from the free-falling and the rebound. When the person jumps, the cord stretches and the jumper flies upwards again as the cord recoils, and continues to oscillate up and down until all the kinetic energy is dissipated.

Questions: a) What is the bungee jumping? b) Can bungee be performed from a movable object? How? c) When do you think bungee becomes thrilling? d) What is the experience when one falls off the platform? e) Where is the bungee jumping point located in India?

Section–IV

Model Question Papers 51

Answer any FOUR of the following: 4x5=20 39. Prepare an attractive advertisement using the hints

given below. Home appliances – Aadi sale – 20 – 50% – special combo offers – Muthusamy & co., Raja Street, Gingee.

40. Write a letter to your friend about Athivarathar Utsavam at Kanchipuram.

41. You are the coordinator of the science forum of your school. An event had been organized on account of National Science Day for the members of the forum. Now, write a report on the observation of “National Science Day” at your school.

42. Look at the following picture and express your views on itinabovefivesentences.

43. Make notes or write a summary of the following passage. My neighborhood is a very interesting place. My home is

located in apartment building downtown near many stores and offices. There is a small supermarket across street, where my family likes to go shopping. There is also post office and bank near our home. In our neighborhood there is a small green park where my friends and I like to play on weekends and holidays. There is small pond near the park and there are many ducks in the park. We always have a great time. In addition there is an elementary school close to our home where my little brother studies in third grade. There are so many things to see and do in my neighborhood. That’s why I like it. It’s really a great place.

X ENGLISH52 IDEAL

44. Identify and correct the errors in the following sentences.

a) As the child fell down so it started crying.

b) He is a honest man.

c) Neither the secretary nor the manager were available.

d) The cat drink milk.

e) One of the chairs are broken.

Section–V

45. Quote from memory. 1x5=5

In the .......... down.

Part–IV 2x8=16

46. Write a paragraph of about 150 words by developing the following hints

a) Miranda – thought – grave – beards – grey – father – delighted – appearance – beautiful - lovely – place – strange – expecting wonders – thought – goddess – island – timidly – simple – enchanted – maid – herself.

(OR)b) Francis Bennet – Editor – World’s largest Newspaper – Earth

Herald – wife – Paris – New planet – Gandhini – aero car – done by machines – everything.

47. a. Read the following paragraph and answer the questions given below:

A patriot is a man who loves his country, works for it, and is willing to fight and die for it. Every soldier is bound to do his duty, but the best soldiers do more than this. They risk their lives because they love the country they are fighting for. They love its hills and valleys, its cities and villages, its people and their way of life and they are willing to defend it to the last against enemies who try to conquer it and destroy it.

Model Question Papers 53

Questions: a) Risk means: i) miss ii) lose iii) endanger iv) meet b) Who is a patriot? c) What do the best soldiers do? d) Why do they risk their lives?

(OR)

Read the following poem and answer the questions given below: And now there came both mist and snow, And it grew wondrous cold: And ice, mast-high, came floating by,As green as emerald.

And through the drifts the snowy cliffsDid send a dismal sheen: Nor shapes of men nor beasts we ken – The ice was all between.

The ice was here, the ice was there, The ice was all around: It cracked and growled, and roared and howled, Like noises in a sound!

At length did cross an Albatross, Thorough the fog it came; As if it had been a Christian soul, We hailed it in God s name.

Questions:a) According to the poet what came there?b) How did the ice look like?c) Where could the poet see the ice?d) Which crossed through the fog?

*****

X ENGLISH54 IDEAL

Key Answers

MODEL QUESTION PAPER - 4

1. c 2. b 3. a 4. d 5. a 6. a 7. b 8. b9. c 10. b 11. c 12. c 13. c 14. c

23. Cricket is being played very well by Radha.24. Shiva requested his friend to give him a hundred rupees.25. ‘What a trouble must I have been to you them!’26. Walk carefully or you will fall down.27. a) Your key note address was informative at the conference. b) I will tie your neck and feet together.28. (i) Go straight and turn right. (ii) Keep walking till the cycle parking area. (iii) Turn left and proceed on Gandhi Road. (iv) Take the second street and you can see the SBI bank at the

corner.44. a) Delete ‘as’ or ‘so’. b) an honest man. c) was available. d) The cat drinks milk. e) is broken.47. a) endanger. b) One who loves his country, works for it and is willing to

fight and die for it. c) They risk their lives. d) Because they love the country.

(OR) a) mist and snow. b) as green as emerald. c) It was all around. d) an albatross.

*****

Model Question Papers 55

MODEL QUESTION PAPER - 5

X - STANDARD

ENGLISHTime: 3.00 hrs Max. Marks: 100

Instructions:

1) Check the question paper for fairness of printing. If there is any lack of fairness inform the Hall supervisor immediately.

2) Use Blue or Black ink to write or underline.

PART –I

i. Answer all the questions. 14x1=14 ii. Choose the most suitable answer and write the code with

corresponding answer.

Choose the appropriate synonyms for the italicised words. 1. I stepped back onto the landing.

a) depressed b) dizzy c) stare d) descend

2. I was amazed to see how well I understand it. a) bored b) explained c) surprised d) expected

3. I asked Aditya whether he was interested I revisiting the place of his birth. a) going over b) leaving c)abandoning d) avoiding

Choose the appropriate antonyms for the italicised words.

4. A retired engraver named Bodwell and his wife. a) affected b) permission c) obliterate d) squeaking

5. Over the third cup of coffee, he glared at Herman and me.a) stared b) smiled c) frowned d) winked

6. I want you to be very attentive. a) cautious b) careful c) considerate d) inattentive

X ENGLISH56 IDEAL

7. Choose the correct plural form of ‘loaf’ from the following: a) lovaes b) loavas c) loaves d) loafes

8. Form the derivative by adding the right prefix to theword–‘Origin’a) – ate b) – un c) – sub d) – dis

9. Choose the correct expansion of the abbreviation ‘AICTE’. a) Along Inform Complete Technical Education b) Access Income Council Technical Education c) All India Council for Technical Education d) All India Credit Technical Educate

10. Complete the following sentence with the most appropriate phrasal verb given below: That helps to ________ the calories. a) burn up b) burn with c) burn off d) burn out

11. Choose the suitable option to pair with the word ‘table’ to form a compound word. a) craft b) spoon c) ground d) coat

12. Fill in the blank with most appropriate preposition given below: ________ the debris, they saw their child.a) Alone b) From c) Because d) Amidst

13. Complete the following sentence using the most appropriate tense form of the given below: Excuse me, I ________ (look) for a hotel.a) looked b) am lookingc) was looking d) is looking

Model Question Papers 57

14. Choose the most appropriate linker from the given four alternatives. I won’t go there ________ you permit me. a) and so b) although c) unless d) but

Part–II (10x2=20)

Section–I

Answer any THREE of the following question in a sentence or two. 3x2=6 15. What do you understand by the mother’s act of throwing the

shoe? 16. Mention the celebrations which the crew enjoyed during their

expedition. 17. What did Aditya offer Sanyal? 18. What instructions did Holmes give Watson to get Mr. Smith?

Section–ll

Read the following sets of poetic lines and answer any THREE of the following: 3x2=6 19. There’s a family nobody likes to meet; They live, it is said, on complaining street

a) Where does the family live? b) Why do you think the street is named as complaining street?

20. At last by starvation and famine made bold All dragging with wet, and all trembling with cold

a) What made the cricket bold? b) Why did the cricket drip and tremble?

21. It never grows leaves Not in the winter, spring, summer or fall it just sits there never getting small or ever growing tall

a) What does ‘it’ refer to? b) In what way is the tree a mystery?

X ENGLISH58 IDEAL

22. We were cast and wrought and hammered to design we were cut and filed and looked and gauged to fit

a) Whom does ‘we’ refer to? b) What do you mean by ‘wrought’?

Section–lll 3x2=6

Answer any THREE of the following: 23. Rewrite the following sentence to the other voice.

Mallika has cooked mutton sauce already.

24. Rewrite using indirect speech. Selvam said, “Wow! What a beautiful picture”.

25. Punctuate the following sentence. may i take zigzag to school papa

26. Transform the following sentence into a complex sentence.At age of six, Varsha started learning music.

27. Rearrange the words in the correct order to make meaningful sentences: a) is dedicated / the teaching community / of / to work /

a majority.b) poetry / can / French / even / recite / he.

Section–IV

Answer the following question: 1x2=2 28. Observe the map given below and write the instructions

required. Guide the Stranger to the Petrol Bunk. Give instructions to help him.

Gan

dhiji

Roa

d

Main Road

Net

haji

Roa

d

SB

ILi

brar

y

Bus stop

Pet

rol

Ban

k

Bha

rath

i N

agar

You are here

Model Question Papers 59

PART–III (10x5=50)

Section–I

Answer any TWO of the following in utmost 10 lines. 2x5=10 29. Describe the funny incident that caused the confusion in the

house. 30. How did Holmes trap Mr. Culverton Smith to confess the murder? 31. Give an account of the last day of M. Hamel at school. 32. How is technology a boon to the disabled? Elucidate your point

of view.

Section–ll

Answer any one TWO of the following in utmost 10 lines. 2x5=10 33. Write a paragraph on “the grumble family” and their attitude

towards other folks. 34. What is the importance of machines in our lives?

35. Read the following stanza and answer the question given below: They growl at the rain and they growl at the sun; In fact, their growling is never done. And if everything pleased them, there isn’t a doubt, They’d growl that they’d nothing to grumble about!

Questions:a) Who does ‘they’ refer to? b) Pick out the rhyming scheme c) What is the figure of speech d) Pick out the alliterated words.

36. Paraphrase the following stanza. Let me but live my life from year to year;With forward face and unreluctant soul Not hurrying to, nor turning from the goalNot mourning for the things that disappear

X ENGLISH60 IDEAL

Section–III

Answer any ONE of the following: 1x5=5

37. Rearrange the following sentences in coherent order. i) Zigzag brought absolute silence in the clinic.

ii) The snoring of zigzag annoyed everyone.

iii) When the fan was switched on, it was raining papayas and bananas.

iv) Dr. Somu asked his cousin Dr. Krishnan to take care of his pet bird Zigzag.

v) It picked up and deposited the fruits on the blades of the fan.

38. Read the following passage and answer the questions that follow.

You are both quite mistaken their father hastened to explain, seeing his wife’s horrified expression. Zigzag is a most harmless, unusual and lovable bird. Apparently, it was bred by a genuine African witch doctor, who gifted it to Somu when he – being a child specialist like me – cured the witch doctor’s son while he was touring the deepest jungles of Equator Africa last month. Somu says the bird is an absolute treasure and a real help. It’s his favourite pet, you know’.

Questions:

a) How was the expression of Mrs. Krishnan, after she heard about zigzag?

b) Who is Zigzag?

c) How was it bred?

d) Who gifted zigzag to Somu?

e) What did Somu say about the bird?

Model Question Papers 61

Section –IV

Answer any FOUR of the following: 4x5=20 39. Prepare an attractive advertisement using the hints

given below. Mobile Galaxy – smart phones – accessories – SIM cards – recharge – free power banks on mobile purchase – No.1, Toll gate Trichy

40. Write a letter to the police inspector to get permission for Pongal celebration in your area.

41. You are the school pupil leader. You have been asked to write a report on the inaugural ceremony of English literary association of your school which was held recently. Write a report in about 120 words.

42. Look at the following picture and express your views on itinabovefivesentences.

43. Make notes or write a summary of the following passage. Books are the greatest treasure of mankind and the habit of

reading them is the source of pleasure. He who is in the habit of reading books should buy books for himself. He should start collecting books in his youth. The books collected and arranged properly in a room not only decorate the room but also make the presence of their authors felt. Books contain in them eternal truths and are better friends than those of flesh and blood as they not only entertain but also guide us.

X ENGLISH62 IDEAL

44. Identify and correct the errors in the following sentences a) Do you know which is world’s tallest building? b) I place great confidence on you. c) He is good athlete who performs well. d) I am absent yesterday. e) I cannot drive as fast as Rahul.

Section –V

45. Quote from memory. 1x5=5 O ‘re rough .......... the best.

Part–IV 2x8=16

46. Write a paragraph of about 150 words by developing the following hints. a) men of 29th century – cold towards everything – compare

their comforts – aero cars – aero buses – population in towns – travel faster – thousand miles an hour – communication – advanced technology – phonotelephote – machines – all kinds of things.

(OR)b) Tom gets a gift – iron box – opening the box – dangerous –

consult educated people – can’t get help – doctor’s advice – spends leisure time in the library – Susan cancels engagement – leaves the box finally.

47. a. Read the following paragraph and answer the question given below:

It took three long years for my coming into existence, after the idea of a new currency was conceived. I was first conceived and named on January, 1999, when eleven European countries decided to have a new money form i.e. Euro. My value was determined as per the conversion rate fixed by these countries in terms of their own currencies. On 1st January, 2002, I was circulated as currency notes when twelve countries adopted me as their currency. Some

Model Question Papers 63

countries opposed the idea of using me and initially refused to accept me. But now almost all the European countries have accepted me as valid money. My brother Dollar who is in America, my brother Lira residing in Italy and my sister Sterling were quite jealous of me, but soon they accepted my existence and started respecting me. I now enjoy a place of pride among all the currencies in the world and almost everyone accepts me freely.

Questions:a) When did Euro come into world wide acceptance? b) Euro was accepted without opposition – Is this statement

true or false? c) Bring out the sibling rivalry in the family of Euro. d) What is the present status of Euro?

(OR)

b. Read the following poem and answer the question given below: A plump little girl and a thin little bird Were out in the meadow together “How cold that poor little bird must be Without any clothes like mine,” said she “Although it is sunshiny weather!” “A nice little girl is that, “said he, But, oh how cold she must be for see She hasn’t a single feather! So each shivered to think of the poor thing Although it was sunshiny weather.

Questions: a) Where were the girl and the bird found? b) What did the girl say about the bird? c) What did the bird think about the girl? d) What was the weather like?

*****

X ENGLISH64 IDEAL

Key Answers

MODEL QUESTION PAPER - 5

1. d 2. c 3. a 4. c 5. d 6. d 7. c 8. a9. c 10. d 11. b 12. d 13. b 14. c

23. Mutton sauce has already been cooked by Mallika.24. Selvam exclaimed that it was a beautiful picture. 25. “May I take Ziazag to school, papa?”26. When Varsha was six, she started learning music.27. a) A majority of the teaching community is dedicated to work. b) He can recite even French poetry.28. (i) Go straight and turn left. (ii) Keep walking till the bus stop. (iii) Take the lane on your left. (iv) After walking past the bus stop, you can see the petrol bunk.44. a) the world’s tallest building. b) in you. c) a good athlete. d) I was absent. e) I cannot drive as rashly a Rahul.47. a) Euro was first conceived and named in January 1999. b) False c) Dollar in America, Lira in Italy and Sterling were the sibling

rivalry in the family of Euro. d) It now enjoys a place of pride among all the currencies in the

world.(OR)

a) in the meadow. b) the bird must be without clothes. c) The bird thought that the girl was without proper clothes. d) sunshine weather.

*****

Model Question Papers 65

MODEL QUESTION PAPER - 6

X - STANDARD

ENGLISHTime: 3.00 hrs Max. Marks: 100

Instructions:

1) Check the question paper for fairness of printing. If there is any lack of fairness inform the Hall supervisor immediately.

2) Use Blue or Black ink to write or underline.

PART –I

i. Answer all the questions. 14x1=14 ii. Choose the most suitable answer and write the code with

corresponding answer.

Choose the appropriate synonyms for the italicized words. 1. He uttered a low cackle.

a) sob b)hark c) moan d) cluck

2. India remained indoors unless permitted. a) effort b) amazed c) captain d) endure

3. I hope you will be able to persuade him to come. a) prevent b) discourage c) repress d) induce

Choose the appropriate antonyms for the italicized words. 4. The project is considered essential towards promoting.

a) covered b) unstudied c) meticulous d) informed

5. His heart stood still. a) motionless b) movingc) restful d) silent

6. The butler appeared at the doorway. a) emerged b) seen c) came out d) vanished

X ENGLISH66 IDEAL

7. Choose the correct plural form of ‘deer’ from the following. a) deers b) deeres c) deer d)

8. Form the derivative by adding the right suffix to theword–‘Hand’.a) big b) gloves c) ful d) write

9. Choose the correct expansion of the abbreviation ‘WHO’. a) World Health Organization b) Wonderful Health Organization c) World Human Organization d) World Health Orientation

10. Complete the following sentence with the most appropriate phrasal verb given below: She says it’s important to ________ good levels of strength and stamina. a) build away b) build up c) build out d) build in

11. Choose the suitable option to pair with the word ‘kitchen’ to form a compound word. a) jug b) spoon c) garden d) house

12. Fill in the blank with most appropriate preposition given below: Everyone talks ________ this across the country. a) above b) about c) to d) from

13. Complete the following sentence using the most appropriate tense form of the given below: The cost of living ________ (rise) very fast. a) was rising b) is rising c) rising d) raised

14. Choose the most appropriate linker from the given four alternatives. The shops remained closed ________ there was a bandh.a) unless b) while c) because d) untill

Model Question Papers 67

Part–II (10x2=20)

Section–I

Answer any THREE of the following question in a sentence or two. 3x2=6 15. What festival did they celebrate during the expedition? 16. What made the young seagull go mad? 17. What were the instructions given by Holmes to Watson? 18. What were the various things that tempted Franz to spend his

day outdoors?

Section–ll

Read the following sets of poetic lines and answer any THREE of the following: 3x2=6

19. Strong is she in her faith and belief. “Persistence is the key to everything,” says she

a) What is she strong about?

b) How does she deal with the adversities in life?

20. Began to complain when he found that, at home His cupboard was empty, and winter was come

a) Whom does ‘He’ refer to?

b) Why was his cupboard empty?

21. Rumours are constantly being made And each day the house just begins to fade What happened inside the house?

a) Does the house remain the same everyday?

b) How does the poet consider the house to be a mystery?

22. And now, if you will set us to our task We will serve you four and twenty hours a day

a) Who will serve us twenty four hours?

b) Explain “Set us to our task”.

X ENGLISH68 IDEAL

Section–lll 3x2=6

Answer any THREE of the following: 23. Rewrite the following sentence to the other voice.

The cables have been removed.

24. Rewrite using indirect speech. cathy left for the united states last sunday.

25. Punctuate the following sentence. only so that you could stay safe

26. Transform the following sentence into a compound sentence.He followed my suggestion.

27. Re arrange the words in the correct order to make meaningful sentences: a) Supportive / were / and / too / they / apprehensive.

b) into action/ are committed / our vision / translating / we / to.

Section–IV

Answer the following question: 1x2=2 28. Observe the map given below and write the instructions

required: Your are waiting for the bus. A middle aged man approaches you seeking help to reach the Cell Park. Guide him in about 100 words.

Mosque

Flower Bazar

Market Road

Big

Baz

ar R

oad

Cell Park

You are here

Model Question Papers 69

Part–III (10x5=50)

Section–I

Answer any TWO of the following in utmost 10 lines. 2x5=10 29. Write in detail about the selection and training process which

the crew underwent. 30. What is the theme of the story “The Night the Ghost got in”?31. What is the message of the lesson “His First Flight”? 32. How did Watson help his friend to arrest the criminal?

Section–ll

Answer any TWO of the following in utmost 10 lines. 2x5=10 33. Critically analyze the poem “No men are foreign”.34. What qualities have made women powerful?

35. Read the following stanza and answer the question given below: She’s a lioness; don’t mess with her. She’ll not spare you if you‘re a prankster. Don’t ever try to saw her pride, her self – respect. She knows how to thaw you, saw you – so beware!

Questions:

a) What is the rhyming word?

b) What is the rhyming scheme?

c) Pick out the words in alliteration.

d) Name the figure of speech used in these lines.

36. Paraphrase the following stanza. ……we who take arms against each otherIt is the human earth that we defileOur hells of fire and dust outrage the innocenceOf air that is everywhere our own.

X ENGLISH70 IDEAL

Section–III

Answer any ONE of the following: 1x5=5 37. Rearrange the following sentences in coherent order.

i) Miranda soon after found Ferdinand almost dying with fatigue.

ii) Miranda hung upon her father and asked him why he was so ungentle.

iii) Ferdinand replied to her that he should finish his task before he took his rest.

iv) She told Ferdinand that he could take rest as her father was at his studies.

v) Prospero had commanded Ferdinand to pile up some heavy logs of wood.

38. Read the following passage and answer the questions that follow.

“Father, listen please,” said Mulan. “For years you trained me in kung fu. You showed me how to use a sword”. Mulan swung the sword back and forth with might. “Only show that you could stay safe!” said her father. “i never meant for you to go to war. If they find out you are a woman, you know as well as I do that you will die!” no one will find out, father, said mulan. She picked up her sword. Mulan said to the father. He tried to get up but had to hold on to his chair. The daughter kissed him goodbye. I love you, father, she said. “Take care of yourself. Tell my brother I said goodbye”. She climbed on a family horse. And off she went to join the emperor’s army.

Questions:a) Who trained Mulan in kung fu? b) How did she swing the sword? c) Did Mulan’s father want her to go to the war? d) What would the emperor do if he found that she was a

woman? e) How did she go to join the army?

Model Question Papers 71

Section –IV

Answer any FOUR of the following: 4x5=20

39. Prepare an attractive advertisement using the hints given below.

MN silks – all varieties – latest collections – low cost – authorized silk mark showroom – festival offer – good customer service.

40. Ms. Latha an NGO writes a letter to the wholesale book shop dealer, placing order for 100 copies of medium size English Oxford Dictionary.

41. You are secretary of Resident Welfare Society, Sundar Avenue, Chennai. Write a circular, which could be sent to all the members of the society to discuss the frequent power cuts and water supply and the ways how to overcome this situation. You are Raghuveersingh.

42. Look at the following picture and express your views on itinabovefivesentences.

43. Make notes or write a summary of the following passage. Discipline is the most essential virtue you should acquire

when you are young. It is a complex quality. It shapes your mind to religious, social and economic patterns when you grow up to be an active citizen. At a young age, we do not realize the value of discipline, as children, we merely long for unlimited freedom, without understanding the meaning of discipline. An

X ENGLISH72 IDEAL

athlete disciplines himself and keeps himself fit. He refrains from smoking, observes good food habits and keeps good health.

44. Identify and correct the errors in the following sentences. a) He is one of the talkative boy in the class. b) Very few students have scored high as Madan. c) Kiran is so ill to go to school. d) Remove the furnitures from here. e) He is an university teacher.

Section –V

45. Quote from memory. 1x5=5 A woman is .......... in spring.

Part–IV 2x8=16

46. Write a paragraph of about 150 words by developing the following hints. a) Mulan – loyal and patriotic – hears the order of the Emperor –

volunteers to fight for her country – joins the army in disguise – wins the war – earns good name – the doctor comes to know – the truth – surprise attack – she tells a plan – she is awarded finally.

(OR)b) Poor farmer – aged mother – order of the despot – farmer

carries his mother – mother’s advice on the way – the son is surprised at her wisdom – takes her back – hides her in the closet – another unreasonable order – mother’s clever idea – a rope of ashes – the governor realises his folly – lifts his decree.

47. a. Read the following paragraph and answer the questions given below:

Some insects are harmful to human beings. The poison of the black widow spider causes unbearable pain. It stiffens the muscles of the abdomen and causes death to some of

Model Question Papers 73

the victims. Some kinds of mosquitoes carry germs that cause diseases such as malaria, yellow fever and sleeping sickness. The mosquitoes pick up the germs and pass them on to the people they bite. Flies play a part in spreading such diseases as cholera, dysentery and typhoid fever. All insects which carry germs and spread diseases live and breed only in dirty places. So we can prevent such diseases if we keep our houses free of dirt.

Questions:

a) How harmful is the bite of the black widow spider?

b) What are the diseases caused by mosquitoes?

c) How do mosquitoes play a part in spreading diseases?

d) Why are flies harmful to us?

(OR)

b. Read the following poem and answer the question given below The sun decending in the west, The evening star does shine, The birds are silent in their nest, And I must seek for mine, The moon, like a flower In heaven’s high bower With silent delight, Sits and smiles on the night

Questions:

a) Where are the birds?

b) What is the time referred to here?

c) What is the moon compared to?

d) Who smiles on the night?

*****

X ENGLISH74 IDEAL

Key Answers

MODEL QUESTION PAPER - 6

1. d 2. d 3. d 4. b 5. b 6. d 7. c 8. c9. a 10. b 11. c 12. b 13. b 14. c

23. Someone has removed the cables.24. Samu asked me if he would come with me.25. Cathy left for the United States last Sunday.26. I suggested something and he followed it.27. a) They were supportive and apprehensive too. b) We are committed to translating our vision into action.28. (i) Go straight and walk across the road. (ii) Walk till you reach the Big Bazar Road on right. (iii) Keep walking till the end of the road. (iv) You can find the cell park opposite the Mosque.44. a) talkative boys. b) as high as. c) too till to go to school d) the furniture. e) a university teacher.47. a) The poison of the black widow spider causes unbearable

pain. b) The diseases caused by mosquitoes are malaria, yellow fever

and sleeping sickness. c) The mosquitoes pick up the germs and pass them on to the

people they bite. d) Flies are harmful to us because they play a part in spreading

such diseases as cholera, dysentery and typhoid fever.(OR)

a) in their nest. c) a flower. b) night. d) the moon.

*****

Model Question Papers 75

MODEL QUESTION PAPER - 7

X - STANDARD

ENGLISHTime: 3.00 hrs Max. Marks: 100

Instructions:

1) Check the question paper for fairness of printing. If there is any lack of fairness inform the Hall supervisor immediately.

2) Use Blue or Black ink to write or underline.

PART-I

i. Answer all the questions. 14x1=14 ii. Choose the most suitable answer and write the code with

corresponding answer.

Choose the appropriate synonyms for the italicised words.

1. Its curved beak was sunflower yellow.

a) timid b) wrong c) rounded d) confirm

2. He lay listless.

a) tried b) happy c) worried d) sad

3. Their father hastened to explain seeing his wife.

a) plenty b) copious c) lifted d) quicken

Choose the appropriate antonyms for the italicised words.

4. He uttered a joyous scream.

a) happy b) sad c) cheerful d) serious

5. The indignant while - haired old man belonged to the house.

a) clam b) angry c) soft d) rough

6. They imprisoned in the bodies of large trees.

a) decide b) freed c) cruelty d) grateful

X ENGLISH76 IDEAL

7. Choose the correct plural form of ‘cupful’ from the following. a) cupsfuls b) cupful c) cupfuls d) cupsful

8. Form the derivative by adding the right prefix to theword–‘scope’.a) extra b) super c) ate d) micro

9. Choose the correct expansion of the abbreviation ‘ALU’. a) Arts Law Unit b) Arithmetic and Logic Unit c) Allover and Legal Unit d) Acquired Lenitive Unknown

10. Complete the following sentence with the most appropriate phrasal verb given below. I ________ the word in the dictionary. a) look away b) look up c) look out d) look in

11. Choose the suitable option to pair with the word ‘baby’ to form a compound word. a) walk b) made c) playing d) sitting

12. Fill in the blank with most appropriate preposition given below. Rahul will return ________ 7 o’ clock. a) after b) about c) before d) on

13. Complete the following sentence using the most appropriate tense form of the given below. The post man ________ (come) everyday at 11 a.m.a) am coming b) is comingc) comes d) will have come

14. Choose the most appropriate linker from the given four alternatives. Call me ________ you need money. a) as soon as b) in case c) while d) despite

Model Question Papers 77

Part–II (10x2=20)

Section–I

Answer any THREE of the following question in a sentence or two. 3x2=6 15. What did the Bodwells think when they heard the mother shout? 16. How long were they trained to undertake this voyage? 17. How was the landscape through which they travelled? 18. What is the future of technology?

Section–ll

Read the following sets of poetic lines and answer any THREE of the following. 3x2=6

19. Besides the house sits a tree It never grows leaves,

a) What is the near the house?

b) Do leaves grow in the tree?

20. Don’t ever try to saw her pride, her self- respect She knows how to thaw you, saw you – so beware!

a) What do the words ‘thaw’ and ‘saw’ mean here?

b) What is the tone of the author?

21. my heart will keep the courage of quest, And hope the road’s last turn will be the best.

a) What kind of quest does the poet seek here?

b) What is the poet’s hope?

22. They too aware of sun and air and water, Are fed by peaceful harvest, by war’s long winter starved

a) What is common for all of us?

b) How are we fed?

X ENGLISH78 IDEAL

Section–lll 3x2=6

Answer any THREE of the following: 23. Rewrite the following sentence to the other voice.

Who has arranged these desks?

24. Rewrite using direct speech. Valar told me that they would complete the work the next week.

25. Punctuate the following sentence. we ll never belong to the family of grumble

26. Transform the following sentence into a compound sentence.Besides being a dancer, she is a singer.

27. Rearrange the words in the correct order to make meaningful sentences. a) my room / out / nobody / there / to clean / is. b) in man’s / plays / role / wealth / today / an important / life.

Section–IV

Answer the following question: 1x2=2 28. Observe the map given below and write the instructions

required:Guidethestrangertothepostoffice.

PETROL BUNK

TRUNK ROAD

H O T E L

Post Office

Yoy are Here

Model Question Papers 79

Part–III (10x5=50)

Section–I

Answer any TWO of the following in utmost 10 lines. 2x5=10 29. Do you agree with Mulan’s decision to go to war? Justify. 30. Write in your own words the various commotions caused by

Zigzag at Dr. Krishnan’s residence. 31. Give an account of the last day of M. Hamel in school. 32. How did Watson help his friend to arrest the criminal?

Section –ll

Answer any TWO of the following in utmost 10 lines. 2x5=10 33. Describe the journey of life as depicted in the poem by Hendry

Van Dyke. 34. Compare and contrast the attitude of the ant and the cricket.

35. Read the following stanza and answer the question given below: They growl at that and they growl at this; Whatever comes, there is something amiss; And whether their station be high or humble, They are all known by the name of grumble.

Questions:a) Pick out the rhyming words.b) What is the rhyming scheme? c) Pick out the words in alliteration. d) Name the figure of speech used in these lines.

36. Paraphrase the following stanza. Let us learn to walk with a smile and a song, No matter if things do sometimes go wrong; And then, be our station high or humble, We ll never belong to the family of grumble

X ENGLISH80 IDEAL

Section –III

Answer any ONE of the following: 1x5=5 37. Rearrange the following sentences in coherent order.

i) The farmer made a rope and presented it as the Governor expected.

ii) He was moved by her love and care for him. iii) A poor farmer and his aged mother lived at the foot of the

mountain. iv) The ruler of the country passed an order to kill all the aged. v) One day the Governor passed an order to the present him

with a rope of ashes

38. Read the following passage and answer the question that follow.

It grew still colder, and his arm ached, began to grow stiff and numb. He shouted again, “Will no one come? Mother! Mother!” but his mother had looked anxiously along the dike road many times since sunset for her little boy, and now she had closed and locked the cottage door, thinking that Peter was spending the night with his blind friend, and that she would scold him in the morning for staying away from home without permission. Peter tried to whistle, but his teeth charted with the cold. He thought of his brother and sister in their warm beds, and of his dear father and mother.

Questions:

a) Why did Peter’s arm ache?

b) What did his mother do?

c) What did she think finally about her son?

d) What did Peter try to do?

e) What did he think?

Model Question Papers 81

Section–IV

Answer any FOUR of the following: 4x5=20

39. Prepare an attractive advertisement using the hints given below.

Zee fine arts academy – admission open – carnatic vocal, veena – violin guitar – keyboard classes – drawing and art classes – contact – no : duration of course – fees.

40. Ramesh wanted to write to Mr. Nagaraj asking him to be the chief guest for a function in his school. Write this letter.

41. You are Suman, secretary, science and technology forum, Chennai. Inform all the students of the school through a notice that there will be some special lecture tomorrow.

42. Look at the following picture and express your views on itinabovefivesentences.

43. Make notes or write a summary of the following passage. The buying and selling of commodities is termed as trade.

People who buy and sell things are called traders. Finished products are bought by traders and sold in markets. People who buy and sell large quantities of commodities are called wholesale merchants. People who buy and sell small quantities are called retailers. When there is surplus of commodities, there is a fall in price. When there is deficit in commodities, there is a rise

X ENGLISH82 IDEAL

in price. The Government supplies essential commodities like rice, dhal, sugar and oil through the enforcement of laws. When there is a shortage of commodities, the Government imports them from other countries and distributes them through fair price shops.

44. Identify and correct the errors in the following sentences.a) Very few people true to their work. b) My brother is three years elder than me. c) Hurry, its getting late. d) He stood with his friend in all difficulties. e) It requires an energy.

Section –V

45. Quote from memory. 1x5=5 From what .......... will be joy.

Part–IV 2x8=16

46. Write a paragraph of about 150 words by developing the following hints. a) Prospero – Duke of Milan – borther Antonio – overthrow

Prospero – king of Naples – Prospero and his daughter – reach an island – releases good spirits – Ferdinand arrives – Prospero tests them – repentance – Propero becomes king again.

(OR)b) men of the 29th century – cold towards everything – compare

their conditions – population in towns – travel faster – aero cars – advanced communication – phonotelephote – machine do all things.

47. a. Read the following paragraph and answer the question given below: Hockey is the national game of India. It had a glorious past in India. The Indian hockey team had won six consecutive

Model Question Papers 83

Olympics. But its present condition is dismal, because hockey is not being given much importance. Still the banner if hockey is held aloft, thanks to the two great players Dhyan Chand and Dhanraj Pillai.

Questions:

a) Which is the national game of India?

b) State the condition of the game in the past.

c) What is the condition of the game now?

d) Who are the two great Indian hockey players mentioned in the passage?

(OR)

b. Read the following poem and answer the question given below: Little drops of water Little grains of sand Make the mighty ocean And the pleasant land Little deeds of kindness, Little words of love, Make the earth and Eden, Like the heaven above And the little moments Humble thoughts they be Make the mighty ages Of eternity,

Questions:

a) What makes the ocean mighty?

b) What kind of words makes the earth?

c) How should be our deeds?

d) Give a suitable title to the poem.

*****

X ENGLISH84 IDEAL

Key Answers

MODEL QUESTION PAPER - 7

1. c 2. a 3. d 4. b 5. a 6. b 7. c 8. d9. b 10. b 11. d 12. c 13. c 14. b

23. By whom have these desks been arranged?24. Varsha said to me, “We will complete the work the next week”.25. We’ll never belong to the family of Gumble.26. She is not only a dancer but also a singer.27. a) There is no body to clean my room out. b) Wealth plays an important role in man’s life.28. (i) Go straight and turn right. (ii) Keep walking along the road. (iii) Take the lane on your right. (iv) Walking a few yards, you can find the post office on left.44. a) are true to their work. b) elder to me. c) Hurry, it’s getting late. d) through all difficulties. e) Delete ‘an’.47. a) Hockey b) The Indian Hockey Team had won six consecutive Olympics. c) Hockey is not given much impotance. d) Dhyan Chand and Dhanraj Pillai.

(OR) a) Little drops. b) Words of love. c) Our deeds should be kind. d) Kindness and Love.

*****

Model Question Papers 85

MODEL QUESTION PAPER - 8

X - STANDARD

ENGLISH

Time: 3.00 hrs Max. Marks: 100

Instructions:

1) Check the question paper for fairness of printing. If there is any lack of fairness inform the Hall supervisor immediately.

2) Use Blue or Black ink to write or underline. PART-I

i. Answer all the questions. 14x1=14 ii. Choose the most suitable answer and write the code with

corresponding answer.

Choose the appropriate synonyms for the italicised words.

1. When I did was seized with fear.

a) left b) caught c) united d) raised

2. I gave out a monstrous cry.

a) horrible b) pleasant c) attractive d) silent

3. I felt dizzy.

a) focus b) giddy c) dirty d) angry

Choose the appropriate antonyms for the italicised words. 4. Cowardice is a baseless.

a) awardees b) courage c) helplessness d) dice

5. Fright seeing some.

a) fear b) freight c) unafraid d) afraid

6. Strange things a few people develop.

a) unusual b) familiar c) unknown d) unwanted

X ENGLISH86 IDEAL

7. Choose the correct plural form of ‘index’ from the following. a) indxes b) indices c) indexs d) index

8. Form the derivative by adding the right prefix to theword–‘ordinary’.a) ultra b) over c) de d) extra

9. Choose the correct expansion of the abbreviation DTH. a) Direct to Home b) Development the Hospital c) Direct to Home Minister d) Danger the Headlight

10. Complete the following sentence with the most appropriate phrasal verb given below: The match was ________ due to heavy rain. a) put away b) put up c) put out d) put off

11. Choose the suitable option to pair with the word ‘made‘ to form a compound word. a) ground b) girl c) prize d) man

12. Fill in the blank with most appropriate preposition given below: She will reach there ________ ten minutes.a) in b) to c) at d) from

13. Complete the following sentence using the most appropriate tense form of the given below: The phone ________ (ring) now. a) rings b) is ringingc) has been rung d) was ringing

14. Choose the most appropriate linker from the given four alternatives. I forget ________ I had to meet the principal. a) that b) because c) until d) despite

Model Question Papers 87

Part–II (10x2=20)

Section–I

Answer any THREE of the following question in a sentence or two. 3x2=6 15. What did the narrator think the unusual sound was? 16. How did the cops manage to enter the locked house? 17. How did the bird feel, when it started flying for the first time? 18. What festival did they celebrate during the expedition?

Section–ll

Read the following sets of poetic lines and answer any THREE of the following. 3x2=6

19. Their hands are ours, and in their lines we read A labour not different from our own.

a) Who does ‘their’ refer to?

b) What does not differ?

20. We can neither love nor pity nor forgive. If you make a slip in handling us, you die!

a) What will happen if you make a slight mistake?

b) What do you mean by the word ‘slip’ here?

21. She today’s woman. Today’s woman dear Love her, respect her, keep her near….

a) Describe today woman according to the poet.

b) How should a woman be treated?

22. Despite the sighs and groans and moans, She’s strong in her faith, firm in her belief!

a) Is the lady complaining about the problem of life?

b) Pick out the words that show her grit.

X ENGLISH88 IDEAL

Section–lll 3x2=6

Answer any THREE of the following:

23. Rewrite the following sentence to the other voice. Their lunch has been prepared and taken by them.

24. Rewrite using direct speech. I ordered him to hold his tongue when she was speaking.

25. Punctuate the following sentence. how big this shop is I wondered.

26. Combine the following sentence into a simple sentence.Ramu is very poor. He cannot buy a bicycle.

27. Rearrange the words in the correct order to make meaningful sentences. a) sister / his / him / parents / praised / and

b) a presentation / to prepare / tomorrow / have / I

Section–IV

Answer the following question: 1x2=2

28. Observe the map given below and write the instructions required: Your uncle visiting from the village wants to go to the ATM and from there to the medical shop. Direct him from your house to his destinations.

2nd Avenue

1st Avenue

ATM

1st S

treet

2nd S

treet

MEDICAL PHARMACY

Your house

Model Question Papers 89

Part–III (10x5=50)

Section–I

Answer any TWO of the following in utmost 10 lines. 2x5=10 29. Describe the struggles undergone by the young seagull to

overcome its fear of flying. 30. Write in detail about the selection and training process which

the crew underwent. 31. How did David’s life change with the use of technology? 32. How did Watson help his friend to arrest the criminal?

Section – ll

Answer any TWO of the following in utmost 10 lines. 2x5=10 33. What is the message of the poem “life”? 34. Write in detail about the selection and training process which

the crew underwent.

35. Read the following stanza and answer the question given below: She’s a lioness; don’t mess with her. She’ll not spare you if you‘re a prankster. Don’t ever try to saw her pride, her self – respect. She knows how to thaw you, saw you – so beware!

Questions:a) Pick out the rhyming words.b) What is the rhyming scheme? c) Pick out the words in alliteration. d) Name the figure of speech used these lines.

36. Paraphrase the following stanza. Don’t ever try to saw her pride, her self – respect. She knows how to thaw you, saw you – so beware! She’s today women. Today’s woman, dear. Love her, respect her, keep her near

X ENGLISH90 IDEAL

Section –III

Answer any ONE of the following: 1x5=5 37. Rearrange the following sentences in coherent order.

i) Though she was sick, she won the final war. ii) So she cut off her hair and went to war as a man. iii) Once the Chinese emperor ordered that one man from each

family must join in the army. iv) She returned to her village with gifts. v) She bravely fought in the war and became the general.

38. Read the following passage and answer the questions that follow.

The eyes of the old mother were not so dim but that they noted the reckless hastening from one path to another, and her loving heart grew anxious. Her son did not know the mountain’s many paths and his return might be one of danger, so she stretched forth her hand and snapping the twigs from bushes as they passed, she quietly dropped a handful every few steps of the way so that they climbed, the narrow path behind them was dotted at frequent intervals with tiny piles of twigs. At last the summit was reached. Weary and heart sick, the youth gently released his burden and silently prepared a place of comfort as his last duty to the loved one. Gathering fallen pine needles, he made a soft cushion and tenderly lifted his old mother onto it. Her wrapped her padded coat more closely about the stooping shoulders and with tearful eyes and an aching heart he said farewell.

Questions:a) How sharp were the eyes of the mother?b) Why did she grow anxious?c) What did she do for her son’s safe return?d) How did he prepare the bed?e) How did he bid farewell?

Model Question Papers 91

Section –IV

Answer any FOUR of the following: 4x5=20

39. Prepare an attractive advertisement using the hints given below. Mouthwatering delicious food – hygienic preparation – affordable price – makes you long for more and more.

40. Imagine that you have parked your two-wheeler inside the school premises. You find it missing. Write a complaint to the head of the school regarding this issue.

41. Riya, a student of class ten, is instructed by her teacher to convey to her classmates about the English literary club competitions which are to be held next week in a nearby government school. Write a notice about it.

42. Look at the following picture and express your views on itinabovefivesentences.

43. Make notes or write a summary of the following passage.

Discipline is the most essential virtue you should acquire when you are young. It is a complex quality. It shapes your mind to religious, social and economic patterns when you grow up to be an active citizen. At a young age, we do not realize the value of discipline, as children, we merely long for unlimited freedom, without understanding the meaning of discipline. An athlete disciplines himself and keeps himself fit. He refrains from smoking, observes good food habits and keeps good health.

X ENGLISH92 IDEAL

44. Identify and correct the errors in the following sentences.

a) My uncle is richest man in the village.

b) Many people behaves rudely now-a-days.

c) A American lives near my house.

d) The dog fell along the river.

e) Each of the cycles are damaged.

Section –V

45. Quote from memory. 1x5=5

Still .......... the best.

Part–IV 2x8=16

46. Write a paragraph of about 150 words by developing the following hints:

a) Dr. Krishanan’s clinic – Dr. Somu – requested shelter for Zigzag – left for Alaska – Zigzag could sing and talk – 21 languages – did not speak – transferred – fruits and nuts – to Chandeller and blades – perched on – curtain rod – slept – fan switched on – fruits and nuts fell down – Mrs. Krishnan’s painting – spoiled – to send the bird away – Dr. Krishnan – took to clinic – Zigzag commanded – brought discipline – the painting – new technique – sold for Rs. 5,000/- – Mr. Krishnan’s family – invited Zigzag – to stay for another week

(OR)

b) 25th july 2889 – Francis Bennet – Managing Editor – Earth Herald – wife – gone to France – switched on Phonotelephote – transmission – saw his wife – went to mechanized dressing room – without attendant’s help – washed – shaved – dressed – went to reporter’s room – enquired – news from his astronomers – new planet – Gandini – travelled to – accumulate work – Niagara – news from his astronomers – got tired – touched a button – take a bath

Model Question Papers 93

47. a. Read the following paragraph and answer the question given below:

Pollution is an undesirable change in physical, chemical and biological characteristics of our land, air and water caused by excessive accumulation of pollutants. The pollutants like Carbon-di-oxide, Sulphur-di-oxide and Nitrogen oxide cause air pollution. The gas Sulphur-di-oxide produces acid rain. Due to acid rain buildings and plant materials are damaged. The increase of these causes global warming also. The average increase in the temperature of atmosphere is called global warming. This is otherwise known as green house effect. This leads to the rise of sea level due to the melting of glaciers. The expansion of water will engulf many parts of this country. July 1998 was the hottest month world over.

Questions:

a) What is pollution?

b) Name a few pollutants.

c) What is global warming?

d) What is the significance of July 1998?

(OR)

b. Read the following poem and answer the question given below:

I often sit a wish that I Could be kite up in the sky And ride upon the breeze and go What – ever way it chanced to below; Then I could look beyond the town, And see the river winding down, And follow all the ships that sail Like me before the merry gate Until at last with them I came To some place with a foreign name.

X ENGLISH94 IDEAL

Questions:a) What does the poet wish? b) What does he wish to ride upon? c) Where would be like to arrive? d) Give the meaning of the word, merry.

*****

Key Answers

MODEL QUESTION PAPER - 8

1. b 2. a 3. b 4. b 5. c 6. b 7. b 8. d9. a 10. d 11. d 12. a 13. b 14. a

23. They have arranged their lunch and taken it.24. I said to him, “Hold your tongue when she is speaking”.25. “How big this shop is !” I wondered.26. Ramu is too poor to buy a bicycle.27. a) His parents praised him and his sister. b) I have to prepare a presentation tomorrow.28. (i) Go straight and cross the road. (ii) Walk till the end of the 1st street and turn right. (iii) Proceed on the Second Avenue. (iv) Ignore the street to your right and walking straight, you can

find the medical pharmacy.44. a) My uncle is the richest man in the village. b) Many people behave rudely now-a-days. c) An American lives near my house. d) The dog fell into the river. e) Each of the cycles is damaged.47. a) Pollution is an undesirable change in physical, chemical and

biological characteristics of our land, air and water. b) Carbon-di-oxide, Sulphur-di-oxide and Nitrogen oxide.

Model Question Papers 95

c) The average increase in the temperature of atmosphere is called global warming.

d) July 1998 was the hottest month the world over.(OR)

a) The poet wishes to be a kite. b) The breeze. c) Some place with a foreign name. d) Happy / Cheerful.

*****

X ENGLISH96 IDEAL

MODEL QUESTION PAPER - 9

X - STANDARD

ENGLISHTime: 3.00 hrs Max. Marks: 100

Instructions:

1) Check the question paper for fairness of printing. If there is any lack of fairness inform the Hall supervisor immediately.

2) Use Blue or Black ink to write or underline.

PART –I

i. Answer all the questions. 14x1=14 ii. Choose the most suitable answer and write the code with

corresponding answer.

Choose the appropriate synonyms for the italicized words.

1. The great expanse of sea stretched down beneath. a) large space b) narrow spacec) small space d) higher area

2. He said in a hopeless tone of a despondent beagle. a) angry b) affluent c) despairing d) strong

3. They were apprehensive and supportive too. a) confident b) inquisitivec) anxious d) special

Choose the appropriate antonym for the italicized words.

4. He said with a groan. a) shout b) growl c) laugh d) cry

5. I had counted on the commotion to get to my desk without being seen ________. a) confusion b) disturbance c) unrest d) calmness

Model Question Papers 97

6. So we can now look forward to a more inclusive way of learning,

a) comprehensive b) completely

c) exclusive d) overall

7. Choose the correct plural form of ‘memorandum’ from the following:

a) memo b) memorandums

c) memoranda d) memoes

8. Formaderivativebyaddingtherightsuffixtotheword–‘reflect’.

a) – by b) – ness c) – ion d) – ly

9. Choose the correct expansion of the abbreviation ‘FDI’.

a) Foreign Direct Impact b) Foreign Digital Investor

c) Foreign Direct Investor d) Foreign Direct Investment

10. Complete the following sentence with the most appropriate phrasal verb given below:

I have managed all my difficulties.

a) got on b) got about c) got in d) got over

11. Choose the suitable option to pair it with the word ‘life’ to form a compound word.

a) weak b) good c) long d) short

12. Fill in the blank with the most appropriate preposition given below:

The Hindu is published ________ Chennai.

a) in b) from c) at d) on

13. Complete the following sentence using the most appropriate tense form of the verb given below:

I ________ (receive) your parcel a few days ago.

a) have received b) received

c) had received d) was receiving

X ENGLISH98 IDEAL

14. Choose the most appropriate linker from the given four alternatives.

Get an entry pass ________ you will not be allowed.

a) or b) otherwise

c) either a or b d) neither a nor b

Part–II (10x2=20)

Section–I

Answer any THREE of the following questions in a sentence or two. 3x2=6 15. How did the bird try to reach its parents without having to fly? 16. Name a few Indian innovations which are helpful to the disabled

and make their day to day life easier. 17. Why was the narrator sorry to have paid attention to the

footsteps? 18. What was the condition of the attic?

Section–II

Read the following sets of poetic lines and answer any THREE of the following. 3x2=6

19. Though our smoke may hide the Heavens from your eyes It will vanish and the stars will shine again

a) What will vanish?

b) Explain the phrase ‘the stars will shine again’.

20. But we ants never borrow We ants never lend

a) What is the reason why ants neither borrow nor lend?

b) Who is the speaker of these lines?

21. I drive past the house almost every day The house seems to be a bit brighter.

a) To whom does “I” refer to?

b) Pick out the words of alliteration in the second line.

Model Question Papers 99

22. They growl at that end and they growl at this Whatever comes, there is something amiss.

a) What does the word ‘growl’ mean here? b) Why do they find everything amiss?

Section–III

Answer any THREE of the following: 3x2=6 23. Rewrite the following sentence to the other voice.

Where have you kept the book?

24. Rewrite using indirect speech. The motorist said to me, “ Can you direct me to the post office?”

25. Punctuate the following sentence. only too well mr smith and he described the symptoms

26. Transform the following sentence into a simple sentence. Though Radhika is poor, she has helping tendency.

27. Rearrange the words in the correct order to make meaningful sentences: a) there / in his manner / was / a certain pride. b) it / storm / two days / before / was / heavy.

Section–IV

Answer the following question: 1x2=2 28. Observe the map given below and write the instructions

required: Your brother needs to go to hospital to visit your sick aunt. Guide him to reach the hospital in about 50 words.

YOU ARE HERE MAIN ROAD

M.G

.Roa

d

School

Hospital

Govt. Boys Hostel

R.R. Super Market

X ENGLISH100 IDEAL

Part–III (10x5=50)

Section–I

Answer any TWO of the following in utmost 10 lines. 2x5=10 29. Describe the last day of M. Hamel in school. 30. Give a detailed account of all thoughts and questions in the

narrator’s mind while accompanying Aditya from the tea shop to Sanyal’s house.

31. How did Watson help his friend to arrest the criminal? 32. How is technology a boon to the disabled?

Section–II

Answer any TWO of the following in utmost 10 lines. 2x5=10 33. What is the central theme of the poem “The Grumble family”? 34. Write a short summary of the poem “The Secret of the Machines”.

35. Read the following stanza and answer the questions given below: We can pull and haul and push and lift and driveWe can print and plough and weave and heat and lightWe can run and race and swim and fly and diveWe can see and hear and count and read and write

Questions:i) Identify the figure of speech employed in the first line. ii) Pick out the rhyming words. iii) Write the rhyming scheme of the given stanza. iv) Find out the poetic device employed in the fourth line.

36. Paraphrase the following stanza. Remember no men are strange, no countries foreignBeneath all uniforms, a single body breathesLike ours: the land our brothers walk uponIs earth like this, in which we all shall lie

Model Question Papers 101

Section–III

Answer any ONE of the following: 1x5=5 37. Rearrange the following sentences in coherent order.

i. Any child in Holland is frightened at the thought of a leak in the dike.

ii. “I am glad they are so strong”, he said to himself. iii. “I’ll stand it somehow” he thought. iv. Peter’s father was one of the men who tended the gates in

the dikes, called sluices.v. If you go quickly, and do not stop to play, you will be home

again before dark” .

38. Read the following passage and answer the questions that follow.

He died that day next week, and was handsomely buried. The day after, his will was found, leaving me his heir. I opened his safe and found in it nothing but an iron box, evidently of his own making, for he was a skilled workman and very ingenious. The box was heavy and strong, about ten inches long, eight inches wide and ten inches high. On it lay a letter to me. It ran thus: I stood appalled, the key in my hand. Was it true? Was it a lie? I had spent all my savings on the funeral, and was poorer than ever. Remembering the old man’s oddity, his malice, his cleverness in mechanic arts, and the patent explosive which had helped to make him rich, I began to feel how very likely it was that he had told the truth in this cruel letter. I carried the iron box away to my lodgings, set it down with care in a closet, laid the key on it, and locked the closet.

Questions:a) When did Uncle die?b) How was Uncle Philip buried?c) What did the author see in the safe?d) On what did the author spend all his savings?e) Where did he carry the iron box?

X ENGLISH102 IDEAL

Section–IV

Answer any FOUR of the following: 4x5=20

39. Prepare an attractive advertisement using the hints given below.

Computer course – three months – flexible timings – discount for bright students – easy EMI.

40. You are Prem / Prema. You live in Chinthamani, Trichy which is adjacent to Cauvery River. You see daily many people throw waste into the river. Write a letter to the Editor of a famous daily, showing your concern about it and also voice your worry. Also state your suggestions to solve the problem.

41. You are Sowmya / Saran, the SPL of GHSS, Villupuram. Prepare a notice about the inauguration of laughter club in your school.

42. Look at the following picture and express your views on itinaboutfivesentences.

43. Make notes or write a summary of the following passage. Languages are of different kinds – spoken language, written

language, sign language, code language, body language, Braille or language of the blind, and more recent ones being the machine language and the computer language. Many languages have a long history. The Greeks, the Romans, the Egyptians the Chinese and the Indians are among those whose languages are centuries old. The number of languages used in

Model Question Papers 103

verbal communication is extremely large. It is said that there are as many as 3500 identified languages throughout the world. Although this is true of the spoken language the number of written languages is much lower, with one estimate placing it at more than 500. Within a language there are sub languages that are called dialects. A dialect is a form of speech peculiar to a district, class or persons. It is a subordinate variety of a language with distinguishable vocabulary, pronunciation or idioms.

44. Identify and correct the errors in the following sentences.

a) Sujeeth jumped over the bore well.

b) If they sing well, they would be selected.

c) We speak the English.

d) A group of tourists has arrived.

e) The evidences are very clear.

Section–V

45. Quote from memory. 1×5=5

By love .......... condemn.

Part–IV

46. Write a paragraph of about 150 words by developing the followinghints. (2x8=16)

a) Francis Bennet – Editor – World’s largest newspaper – Earth Herald – Paris – New plant – Gandhini – aerocar – done by machines – everything.

(OR)

b) Holland – land – below sea level – dikes protected the country – everyone did best to-protect – Holland – Years ago – little – boy Peter – lived Holland – His father – attended – dyke gates – opened – closed dykes – one day – Peter mother – gave cakes to Peter – to be given – old blind friend of Peter – across the dyke – Peter happily left home – Peter visited – old

X ENGLISH104 IDEAL

man – returned near by the dyke – heard – water trickling – stopped to see – small hole dyke – called for help – in vain – he put his little – finger – throughout the night – slept near the dyke – morning – found by passer by – alerted the people – Peter and Holland – saved.

47. Read the following paragraph and answer the questions given below:

Garbage is a great environmental hazard. It comes from various sources – used paper, tiff in packing, plastic bags, ice – cream wrappers, bottle caps, fallen leaves from trees and many more, Garbage makes the premises ugly, unkempt and breeds diseases. A lot of trash that is thrown away contains material that can be recycled and reused such as paper, metals and glass which can be sent to the nearest recycling centre or disposed of to the junk dealer. It also contains organic matter such as leaves which can enrich soil fertility. A compost pit can be made at a convenient location where the refuse can be placed with layers of soil and an occasional sprinkling of water. This would help decomposition to make valuable fertilizer. This would also prevent pollution that is usually caused by burning such organic waste.

Questions:

a) What does the word “unkempt’ mean?

i) having a neat appearance

ii) having untidy appearance

iii) being decent

iv) being indecent

b) Pick out the American English word used in the passage.

c) State whether the following statement is True or False.

Glass can be recycled.

d) What would help decomposition to make valuable fertilizer?

Model Question Papers 105

(OR)

Read the following poem and answer the questions given below:

I’m an angel disguised with dimpled cheeks and laughing eyes.

Don’t you want me? I am your baby.

I have come as a gift from heaven’s hall in your heart.

Oh, hear my call Mother keep me I am your baby

Oh Mother, let me live, don’t take away my life. Mother let me live.

You know it isn’t right to stop me being born, I want to be yours

Oh Mother, let me live, don’t take away my life. Mother let me live.

I want to live my life.

Mother, you will see when you look at me and you hold me in your arms

You’ll fall In love with me,

Like a flower in your care, I am a gift so pure and fair.

Don’t you want me? I am your baby.

Questions:a) Whose cry is this poem?b) Why is the speaker in danger?c) What does the mother know?d) Which expression expresses the child’s desire to live?

*****

X ENGLISH106 IDEAL

Key Answers

MODEL QUESTION PAPER - 9

1. a 2. c 3. c 4. c 5. d 6. c 7. c 8. c9. d 10. d 11. c 12. a 13. b 14. c

23. Where has the book been kept by you?24. The motorist asked me if I could direct him to the post office.25. “Only too well, Mr. Smith”, and he described the symptoms.26. In spite of her poverty, Radhika has helping tendency.27. a) There was a certain pride in his manner. b) It was a heavy storm two days before.28. (i) Go straight and ignore M.G road on right. (ii) Turn right. (iii) Go past the school. (iv) You can see the hospital on your right.44. a) Jumped into the borewell b) They will be selected. c) Delete ‘the’ d) have arrived. e) The evidence.47. a) (ii) b) trash. c) True. d) A composite bit.

(OR) a) a baby. b) afraid of life being taken away. c) Mother knows that it is her baby. d) I want to live my life.

*****

Model Question Papers 107

MODEL QUESTION PAPER - 10

X - STANDARD

ENGLISHTime: 3.00 hrs Max. Marks: 100

Instructions:

1) Check the question paper for fairness of printing. If there is any lack of fairness inform the Hall supervisor immediately.

2) Use Blue or Black ink to write or underline. PART –I

i. Answer all the questions. 14x1=14 ii. Choose the most suitable answer and write the code with

corresponding answer.

Choose the appropriate synonyms for the italicized words.

1. Father had left the ancestral home and moved to Kolkata.

a) interested b) inherited c) flexible d) devoted

2. I sat in silent dejection ________.

a) delight b) anger c) calmness d) hopelessness

3. Uncle reminded him to go home.

a) partly b) advise c) recall d) jealous

Choose the appropriate antonyms for the italicized words.

4. At regular intervals a board creaked.

a) poor b) common c) irregular d) crazy

5. The attic had crumbled down.

a) blessed b) revealed c) consolidate d) incurious

6. Instantly the steps began again.

a) gradually b) rapidly c) calmly d) modernly

X ENGLISH108 IDEAL

7. Choose the correct plural form of ‘radius’ from the following. a) radio b) radii c) radis d) radios

8. Form the derivative by adding the right suffix to theword–‘establish’.a) – able b) – our c) – ly d) – ment

9. Choosethecorrectexpansionoftheabbreviation‘CCTV’.a) Closed Control Televisionb) Closed Circuit Televisionc) Closed Conclusion Televisiond) Control Circuit Television

10. Complete the following sentence with the most appropriate phrasal verb given below: I could not ________ with the others since I am a slow learner.a) keep up b) keep off c) keep up d) keep out

11. Choose the suitable option to pair with the word ‘write’ to form a compound word. a) up b) on c) with d) for

12. Fill in the blank with most appropriate preposition given below: The politician spoke ________ Gandhian philosophy. a) above b) about c) against d) still

13. Complete the following sentences using the most appropriate tense form of the given below: She ________ (wear) a new dress now. a) is wearing b) wearsc) was wearing d) are wearing

14. Choose the most appropriate linker from the given four alternatives.On Friday night we watched a drama ________ a movie.a) though b) and c) but d) even

Model Question Papers 109

Part–II (10x2=20)

Section–I

Answer any THREE of the following question in a sentence or two. 3x2=6 15. What do you understand by the mother’s act of throwing the

shoe? 16. Who were Aditya’s ancestors?17. Mention the celebrations which the crew enjoyed during their

expedition.18. Why does M. Hamel say that we must guard our language?

Section–ll

Read the following sets of poetic lines and answer any THREE of the following. 3x2=6

19. At last by starvation and famine made bold, All dripping with wet, and all trembling with cold,

a) What made the cricket bold?

b) Why did the cricket drip and tremble?

20. I drive past the house almost everyday The house seems to be a bit brighter

a) Whom does ‘I’ refer to?

b) Pick out the words of alliteration in the second line.

21. My heart will keep the courage of the quest And hope the road’s last turn will be the best

a) What kind of quest does the poet seek here?

b) What is the hope of the poet?

22. They growl at that and they growl at this Whatever comes, there is something amiss

a) What do you mean by ‘growl’?

b) Why do they find everything amiss?

X ENGLISH110 IDEAL

Section–lll 3x2=6

Answer any THREE of the following:

23. Rewrite the following sentence to the other voice.

Nobody has answered the questions.

24. Rewrite using direct speech.

Kathir exclaimed joyfully that his brother got the certificate.

25. Punctuate the following sentences into a complex sentence.

alas alas cried the farmer

26. Combine the following sentence into a complex sentence.

Latha saw a snake. At once she ran away.

27. Rearrange the words in the correct order to make meaningful sentences.

a) a result / careful planning / is / and execution / success / of.

b) speaks / Raga / besides / German / languages / two.

Section–IV

Answer the following question: 1x2=2

28. A strange wants to visit the KPN House Write the steps to guide him to reach his destination.

Ammankovil Street

KPN HouseTemple

Model Question Papers 111

Part–III (10x5=50)

Section–I

Answer any TWO of the following in utmost 10 lines. 2x5=1029. Man does change with time – what were the various changes

that came about in Aditya? 30. Describe the last day of M. Hamel at school.31. Narrate the extensive search operation made by the policeman

in the house.32. How did Holmes trap Smith to confess the murder?

Section–ll

Answer any TWO of the following in utmost 10 lines. 2x5=1033. Compare and contrast the attitude of the ant and the cricket?

34. What kind of behavior does the poet want the readers to possess from the poem “The Grumble Family”?

35. Read the following stanza and answer the question given below:

A silly young cricket, accustomed to sing Through the warm, sunny months of gay summer and spring, Began to complain when he found that, at home, His cupboard was empty, and winter was come.

Questions:

a) Pick out the rhyming words.

b) What is the rhyming scheme?

c) What is the figure of speech?

d) Pick out the alliterated words.

X ENGLISH112 IDEAL

36. Paraphrase the following stanza.

She’s lioness; don’t mess with herShe’ll not spare you if you’re a pranksterDon’t ever try to saw her pride, her self respect She knows how to thaw you, saw you – so beware

Section–III

Answer any ONE of the following: 1x5=5

37. Rearrange the following sentences in coherent order. i) The son made up his mind to take back his mother home.

ii) A farmer decided to leave his old mother on top of a mountain.

iii) Once in Shinning, a cruel ruler declared that all old people must be put to death.

iv) When the farmer bade farewell, she advised him to return home with the aid of twigs.

v) The mother dropped the small twigs as markers on the way to help her son return.

38. Read the following passage and answer the question that follow.

Many years ago, China was in the middle of a great war. The Emperor said that one man from each Chinese family must leave his family to join the army. Mulan, a teenage girl who lived in a faraway village of China, heard the news when she was outside, washing clothes. Mulan ran into the house. Her father was sitting in a chair, carving a piece of wood. “Father!” she said. “Did you hear what the Emperor says each family must do?” “Yes,” said her old father, “I heard about it in town. “Your brother is a child. He cannot go.” “Of course that’s true,” said Mulan. “He is too little. But I have an idea.” She poured her father a cup of tea and handed it to him. “Father, have some tea. I will be right back.” Mulan went into her room. With her sword, she cut off her long, black hair. She put on her father’s robe. Going back to her

Model Question Papers 113

father, Mulan said, “Look at me. I am your son now. I will go in your place. I will do my part for China.” “No, my daughter!” said the old man. “You cannot do this!”

Questions:a) Which country does Mulan belong to?b) What was Mulan doing when she heard the news?c) What was Mulan’s father doing when Mulan came to deliver

the news?d) Where did Mulan’s father hear the news?c) Why did Mulan want to join the army?

Section–IV

Answer any FOUR of the following: 4x5=20

39. Prepare an attractive advertisement using the hints given below. Fresh fruits – vegetables from farms – healthy – juicy – tasty – low price – care for health – fruits and vegetables – 59, Mint street, Trichy.

40. Write a letter to your headmaster requesting to issue a transfer certificate to join an ITI course.

41. You are the school monitor, of Modern Matriculation School, Villupuram. Your school principal has requested you to inform the students about a trip to Yercaud for 3 days. Prepare a notice giving the details such as date of journey, mode of transportation, amount, dress code etc.

42. Look at the following picture and express your views on itinabovefivesentences.

X ENGLISH114 IDEAL

43. Make notes or write a summary of the following passage. Moral courage is of the mind, while physical courage is of the

body; but mind an body are so closely connected that I think it is impossible to separate the one sort of courage from the other. In many instances of human daring, moral courage combined with physical to a resolute endurance of physical suffering becomes added to a consciousness of duty. But while physical courage in a human being implies at the least, the moral courage implies much more. This moral courage is the courage which braces us always to do our duty. This is the noblest form of courage and yet it is the courage most commonly required. It is the courage which everyday, almost every hour, demands of us all.

44. Identify and correct the errors in the following sentences.a) He is the good batsman in our team. b) Economics are my favorite subject. c) The tree is very tall that we cannot climb upon it. d) He is angry on me. e) One of the boy has won a prize.

Section–V

45. Quote from memory. 1x5=5 Let me .......... and moans.

Part–IV 2x8=16

46. Write a paragraph of about 150 words by developing the following hints. a) Dr. Krishanan’s family – Dr. Somu – requested for shelter

– the Zigzag – left for Alaska – Zigzag could sing and talk – 21 languages – did not speak – transferred – fruits and nuts – to chandlier and blades – perched on – curtain rod – slept – fan switched on – fruits and nuts fell down – Mrs. Krishnan’s painting spoiled completely – got angry – get rid of the zigzag – Dr. Krishnan – took it to clinic – Zigzag commanded – brought discipline – the painting – new technique – sold for

Model Question Papers 115

Rs. 5000 – Mr. Krishnan’s family – invited zigzag – to stay for another week

(OR)b) Uncle Philip – inventor – mechanical engineer – lonely life –

mania for collection of precious stones – called Tom – told a strange story – Tom as heir – pay for funeral – left of box – full of precious stones – died after a week – letter on the box – box with lot of rubies – diamonds and pearls – also dynamite – if opened with doubt – would explode – Tom started thinking – ways to open – consulted Professor Clinch – no solution – read about explosives – suspicious – changed his name and occupation – at last – gave the box – to the charitable society.

47. a. Read the following paragraph and answer the question given below: Our Earth is the only planet with a variety of plants, animals and the micro organisms living either in the acquatic or the terrestrial habitats. An environment with rich biological diversity is the basis for human existence. The equilibrium status of living and non living factors in an environment is known as balance in nature. It is disturbed at times due to natural calamities like storm, flood, pests, outbreak of diseases and fire. Rapid population growth, industrialization and urbanization are the causes for deforestation exploiting fauna and flora against eco system. The Indian Parliament has passed the Wild life Protection Act in 1980 to preserve and protect our natural wealth.

Questions:a) Which the basis for human existence? b) How is balance in nature established? c) Mention some natural calamities that disturb the balance

in nature. d) Which Act has been passed in our country to protect the

forests? When was it passed?

X ENGLISH116 IDEAL

(OR)

b. Read the following poem and answer the question given below

I was angry with my friend;I told my wrath, my wrath did end.

I was angry with my foe;I told it not, my wrath did grow.

And I water’d it in fears,Night and morning with my tears;

And I sunned it with smiles,And with soft deceitful wiles.

And it grew day and night,Fill it bore an apple bright;

And my foe behold it shineAnd he knew that it was mine.

And into my garden stoleWhen the night had veil’d the pole;

In the morning glad I seeMy foe outstretched beneath the tree.

Questions:

a) What happened when the poet did not show his anger?

b) How did the poet water his anger?

c) What did the poet’s enemy behold?

d) What did the poet find in the garden in the morning?

*****

Model Question Papers 117

Key Answers

MODEL QUESTION PAPER - 10

1. b 2. d 3. c 4. c 5. c 6. a 7. b 8. d9. b 10. a 11. a 12. b 13. a 14. b

23. These questions have not been answered by anyone.24. Kathir said “Oh! my brother has got the certificate.”25. “Alas! Alas!” cried the farmer.26. As soon as Latha saw a snake, she ran away.27. a) Success is a result of careful planning and execution. b) Besides two languages, Raja speaks German.28. (i) Go straight. (ii) Take the lane on your light. (iii) Keep walking till the end of the lane. (iv) You can see KPN House on right at end of the road.44. a) a good batsman of out team. b) Economics is. c) The tree is so tall. d) angry with me. e) One of the boys.47. a) An environment with rich biological diversity is the basis for

human existence. b) Balance in nature is established by maintaining the

equilibrium status of living and non living factors. c) storm, flood, pests, outbreak of diseases and fire. d) The Wild life Protection Act has been passed. It was passed

in 1980.(OR)

a) His anger grew. b) He watered his anger in fears and tears. c) He beheld an apple in the garden that stands for anger. d) The poet found his enemy dead beneath the tree.

*****

X ENGLISH118 IDEAL

NOTES

*****